Ich war erst letzte Woche zu Besuch am Europäischen Kernforschungszentrum CERN und hab mich über die aktuelle Teilchenphysik informiert (auch wenn ich die Mathematik des Standardmodells der Teilchenphysik nicht komplett verstanden habe).

Aber eines habe ich auf jeden Fall verstanden: Es gibt stabile Teilchen und instabile Teilchen. Stabil ist das Zeug, aus dem wir bestehen: also zum Beispiel Elektronen und die Up- bzw. Down-Quarks aus denen die Protonen und Neutronen aufgebaut sind (die wiederum die Atomkerne der Materie bilden). Diese Teilchen existieren unendlich lange. Ein Elektron bleibt für immer und ewig ein Elektron und ändert sich nicht. Andere Teilchen aber sind instabil. Zum Beispiel die restlichen vier Quarks die neben Up und Down noch existieren (Charm, Strange, Top und Bottom). Sie leben nur Sekunden oder Sekundenbruchteile, bevor sie zerfallen und aus der freiwerdenen Energie neue Teilchen entstehen, die dann selbst wieder zerfallen; so lange, bis am Ende wieder die normalen und stabilen Teilchen übrig bleiben. Manche Teilchen überleben auch nur in Gesellschaft. Ein einsames Neutron bleibt nur für wenige Minuten stabil, bevor es zerfällt – ist es dagegen als Teil eines Atomkerns an ein Proton gebunden, dann bleibt es dauerhaft stabil.

Auch das Licht kann in der Quantenmechanik als Teilchen behandelt werden. Es besteht aus Photonen und diese Lichtteilchen müssten dauerhaft stabil sein. Immerhin haben sie keine Masse und wenn etwas keine Masse hat, kann es auch nicht in andere Teilchen zerfallen. Aber was, wenn das Photon doch eine Masse hat? Dann könnte es zerfallen – das Licht selbst wäre instabil. Aber das ist doch sicher nur Unsinn – wie soll und kann Licht Masse haben?

Julian Heeck vom Max-Planck-Institut für Kernphysik in Heidelberg hält diese Frage nicht für Unsinn und hat eine wissenschaftliche Arbeit mit dem Titel “How stable is the photon?” veröffentlicht. Darin erklärt er, dass es zwar stimmt, dass das Photon in den quantenmechanischen Gleichungen die sein Verhalten beschreiben, keine Masse hat und dass die Gleichungen nicht mehr funktionieren, wenn man dem Photon eine Masse gibt. Aber es gibt Möglichkeiten, diese Gleichungen so umzuformulieren, dass sie weiterhin so funktionieren wie vorher, nur dass diesmal das Photon nicht zwingend masselos sein muss.

Auch das Mondlicht könnte instabil sein...

Auch das Mondlicht könnte instabil sein…

Die Theorie erlaubt also, dass Licht eine Masse hat und wenn das theoretisch möglich ist, dann gibt es keinen Grund, warum man einfach so von masselosen Photonen ausgehen sollte, meint Julian Heeck. Natürlich kann die Masse der Photonen nicht sonderlich groß sein; das hätten wir dann schon längst gemessen. Aber vielleicht sind die Photonen so ähnlich wie die Neutrinos. Das sind ebenfalls Elementarteilchen, die eine so geringe Masse haben, dass man sie sehr lange für masselos gehalten hat. Erst seit ein paar Jahren reichen die experimentellen Daten aus, um sicher sein zu können, dass die Neutrinos nicht masselos sind.

Heeck schlägt also vor, die potentielle Masse und damit auch die Lebensdauer eines Photons anhand von Beobachtungsdaten einzuschränken. Dazu benutzt er die Daten des Planck-Satelliten. Was der macht und misst habe ich in diesem Artikel ausführlich erklärt. Planck beobachtet die kosmische Hintergrundstrahlung; das sind Photonen, die kurz nach dem Urknall selbst entstanden sind und sich seitdem durch das All bewegen. Es ist das älteste Licht das wir sehen können und wenn man auf der Suche nach zerfallenden Photonen ist, muss man sein Glück dort versuchen. Wenn Photonen tatsächlich irgendwann instabil werden, dann sollten auch ein Teil des kosmischen Hintergrundlichts zerfallen. Wir sollten heute weniger Photonen messen können als damals vor 13,8 Milliarden ausgesandt wurden und um so weniger, je kürzer die durchschnittliche Lebenszeit der Photonen ist.

Man muss also die Beobachtungsdaten von Planck mit den theoretischen Vorhersagen vergleichen und nachsehen, ob es da Unterschiede gibt. Und wie ich im oben erwähnten Artikel erklärt habe, stimmen Plancks Beobachtungen mit den Vorhersagen extrem gut überein. Die Masse kann also wirklich nur klein sein und wenn man den größtmöglichen Wert nimmt, der noch mit den Beobachtungsdaten in Einklang zu bringen ist, dann folgt daraus eine Lebenszeit für das Photon von drei Jahren. Das klingt überraschend wenig, bezieht sich aber nur auf das Ruhesystem des Photons. Wenn man wissen will, wie lang das Photon aus Sicht eines externen Beobachters überlebt, muss man die relativistische Zeitdilatation berücksichtigen. Und Photonen sind schnell; das Licht bewegt sich immerhin mit Lichtgeschwindigkeit. Für uns lebt das Photon daher ungefähr eine Trillion Jahre – was deutlich länger ist als die bisherige Lebensdauer des Universums (knapp eine Milliarde mal länger!). Diese Zahl ist natürlich nur ein statistischer Mittelwert. Es wird Photonen geben, die wesentlich länger leben und Photonen, die schon viel früher zerfallen. Trotzdem ist das Photon immer noch ein enorm stabiles Teilchen – aber es könnte eben ein Teilchen sein, das nicht dauerhaft stabil ist.

Heeck schreibt am Ende seiner Arbeit:

“In conclusion, a massive photon sounds crazy and exotic, but it really is not. A massless photon is neither a theoretical prediction nor a necessity, but rather a phenomenological curiosity.”

Licht mit Masse klingt tatsächlich ein wenig verrückt – aber man muss sich eben auch mit den verrückten Dingen beschäftigen, solange sie nicht im Widerspruch zur restlichen Physik und den Beobachtungen stehen. Und bei instabilen Photonen ist das der Fall. Heeck schlägt für zukünftige Arbeiten ausführliche theoretische Untersuchungen der Eigenschaften des frühen Universums vor, um vielleicht noch bessere Vorhersagen zum Vergleich mit den Beobachtungen machen zu können. Wer weiß, was dabei heraus kommen wird…

(Und wenn das Thema mal bis zu den normalen Medien durchdringt, werden wir sicher bald mit Schlagzeilen der Form “Skandal! Zerfallende Photonen: Wissenschaftler machen das Licht radioaktiv!” rechnen müssen…)

via Backreaction

Kommentare (129)

  1. #1 Dietmar
    29. Juli 2013

    Wenn das Photon eine, auch noch so kleine Masse hat, dann müsste daraus doch folgen, dass die Lichtgeschwindigkeit höher wäre, wenn das Photon masselos wäre. Es wäre also vielleicht denkbar, dass es unentdeckte masselose Teilchen gibt (vielleicht Teilchen, die die dunkle Energie ausmachen), die schneller als das Licht wären?

    (Ich hoffe ja stark, das war eine hinreichend mit Konjunktiven durchsetzte Frage … 🙂 )

  2. #2 Dietmar
    29. Juli 2013

    “Wenn das Photon eine, auch noch so kleine, Masse hat”

  3. #3 Ferraristo
    29. Juli 2013

    ähm
    da die Photonen mit Lichtgeschwindigkeit reisen. dürften sie überhauptnicht altern.
    Die Zeit bleibt bei Lichtgeschwindigkeit ja “stehen”.
    Jedenfalls hab ich es so gelernt
    Oder Widerspricht mir jetzt jemand?

  4. #4 Dietmar
    29. Juli 2013

    @Ferraristo: Ja, aber wenn sie Masse haben, ist die Lichtgeschwindigkeit nicht die “tatsächliche” Lichtgeschwindigkeit, bei der das gilt; sie müssten etwas, nur minimal, aber etwas langsamer sein, als die “echte” Lichtgeschwindigkeit.

  5. #5 Florian Freistetter
    29. Juli 2013

    @Ferraristo, Dietmar: “Wenn das Photon eine, auch noch so kleine Masse hat, dann müsste daraus doch folgen, dass die Lichtgeschwindigkeit höher wäre, wenn das Photon masselos wäre. Es wäre also vielleicht denkbar, dass es unentdeckte masselose Teilchen gibt (vielleicht Teilchen, die die dunkle Energie ausmachen), die schneller als das Licht wären?”

    Richtig – “Lichtgeschwindigkeit” ist auch ein irreführender Begriff. Es gibt ja noch viel mehr Zeugs das sich mit dieser Geschwindigkeit bewegt. Man sollte es “Geschwindigkeit von masselosen Teilchen” nennen. Und wenn das Licht Masse hat, wäre es eben ein klein wenig langsamer…

  6. #6 Ferraristo
    29. Juli 2013

    @FF
    @Dietmar
    da merkt man schon dass, ich gerade aufgestanden bin
    Klar, ein Teilchen mit Masse kann sich ja nicht mit LG bewwegen.
    Ein Denkfehler von mir
    aber trotzdem danke 😀

  7. #7 kamikaze
    Karlsruhe
    29. Juli 2013

    Mich verwirrt das ganze. Ich bin nie auf die Idee gekommen, dass Photonen /keine/ Masse haben. Schließlich können wir Phänomene wie Gravitationslinsen und schwarze Löscher beobachten. Wie sollte Gravitation auf masselose Teilchen wirken?

  8. #8 Florian Freistetter
    29. Juli 2013

    @kamikaze: “Wie sollte Gravitation auf masselose Teilchen wirken?”

    Weil Newtons Formel mit “Masse1 * Masse2” nicht unbedingt die Realität wiederspiegelt, sondern nur ein Modell ist, dann nicht überall funktioniert. Einstein erklärt die Gravitation durch die Krümmung des Raums. Und ein Lichtstrahl folgt einfach dem kürzesten Weg durch die Raumzeit. Wenn nun also eine Masse den Raum krümmt, wird das Licht (und alles andere auch) dieser Krümmung folgen und dadurch abgelenkt. Das ganze sieht dann so aus, als würde die Masse das Licht anziehen.

  9. #9 Bullet
    29. Juli 2013

    hmmm … allerdings frage ich mich, was denn die Bruchstücke sind, in die Photonen zerfallen können. Noch dazu, wo Photonen ja Bosonen mit dem Spin 1 sind und daher … öh … einige Haken, an die man sich hängen kann, gerade nicht aufweisen…

  10. #10 Florian Freistetter
    29. Juli 2013

    @Bullet: “llerdings frage ich mich, was denn die Bruchstücke sind, in die Photonen zerfallen können.”

    Vielleicht zerfallen die in DUNKLES LICHT!!! mit dem man dann die dunkle Materie sehen kann?

    Ne, ich hab keine Ahnung. Das muss man Teilchenphysiker fragen…

  11. #11 kingchaos
    Karlsruhe
    29. Juli 2013

    Interessanter Gedanke auf jeden Fall.

    In dem Paper wird wohl unter anderem der Zerfallskanal in zwei Neutrinos vorgeschlagen. Nun ja, je nach Massenverhältnissen eben. Müsste man aber wirklich mal einen Teilchenphysiker fragen…

  12. #12 ChristianS
    29. Juli 2013

    Warum soll etwas ohne Masse nicht zerfallen können?

  13. #13 Bullet
    29. Juli 2013

    Hm. Vielleicht weil dann eine der Hauptmotivationen des Zerfalls nicht gilt?

  14. #14 Heute mal anonym
    29. Juli 2013

    Zum Foto: Wie soll das Mondlicht denn auch heile bleiben, wenn es aus so großer Höhe ins Wasser fällt?

  15. #15 Pyrex
    29. Juli 2013

    Interessante Vorstellung! Wenn Photonen nun Massse besitzen würde, wäre dann deutlich mehr Masse im Universum vorhanden oder wäre die Menge vernachlässigbar gering?

  16. #16 Jarrus Poritschew
    29. Juli 2013

    Verzeiht einem Physiklaien..:
    Ich bin auch immer davon ausgegangen, dass Photonen, zumindest in Bewegung (& für stillstehende Photonen fällt mir kein Beispiel ein) eine Masse haben. Ich erinnere mich noch an das Experiment, wo ein Laser auf einen freihängenden Spiegel schoß und der Spiegel nach hinten schwang. Ohne Masse kein Rückstoß war meine Folgerung.
    Und was ist mit E=mc²? Bei m=0 ist E auch 0…?

  17. #17 Florian Freistetter
    29. Juli 2013

    @Jarrus: Das mit dem Spiegel und dem “Rückstoss” hat nicht so sehr mit Masse zu tun, sondern mit dem Impuls. Und den haben auch Photonen.

  18. #18 Conz
    29. Juli 2013

    Einfach das Licht abbremsen, dann werden aus den Trillion Jahren die postulierten drei Jahre, und damit beobachtbar.
    https://www.dradio.de/dlf/sendungen/forschak/2178838/
    Ok, die Forscher sind “erst” bei einer Minute, aber sind auf der Suche nach Kristallen, die deutlich längere “Speicherzeiten” haben könnten.
    Damit sollte es sich doch nachweisen lassen, ob Photonen zerfallen oder nicht.

  19. #19 Alderamin
    29. Juli 2013

    @Pyrex

    Wenn Photonen nun Massse besitzen würde, wäre dann deutlich mehr Masse im Universum vorhanden oder wäre die Menge vernachlässigbar gering?

    Die Gesamtmasse, die die Strahlung im Weltall beiträgt, sollte eigentlich bekannt sein. Die Frage ist, wieviel davon auf eine potenzielle Ruhmasse entfällt, und wieviel bewegte Masse ist. Bisher gilt: 0% zu 100%.

  20. #20 Silava
    29. Juli 2013

    @ChristianS: Warum soll etwas ohne Masse nicht zerfallen können?

    Ganz einfach: Ein Teilchen ohne Masse ist immer mit Lichtgeschwindigkeit unterwegs. Und wer sich mit Lichtgeschwindigkeit bewegt, für den vergeht die Zeit nicht. Ein masseloses Teilchen hat also gar keine Zeit um zu zerfallen. Das hat Ferraristo schon bei #3 geschrieben.

  21. #21 ChristianS
    29. Juli 2013

    @Silava

    Danke. Im Artikel war es so beiläufig erwähnt, als sei gemeint: da ist ja nichts zum Zerfallen.
    Und bei #3 habe ich nicht geschaltet.

  22. #22 maunzz
    kosmos
    29. Juli 2013

    “Können Photonen zerfallen?”

    Und in was sollen sie zerfallen ? In 2 Neutralinos oder was ?

  23. #23 Dr. Feelgood
    Bavaria
    29. Juli 2013

    Hmmm, ich sehe schon im Voraus, wie die hier versammelte Gemeinde von physikaffinen Zeitgenossen über mich als stillen, jedoch durchaus interessierten Mitleser herfallen wird, wenn ich zu dem Thema ausnahmsweise mal meinen Senf abgebe, bzw. eine provokante Frage in den Raum stelle: könnte der hier angesprochene Zerfall im Lauf von Jahrmilliarden nicht der wahre Grund für die beobachtete Rotverschiebeung im Lichtspektrum von sehr weit entfernten Objekten sein?

    So, jetzt dürft Ihr auf mich einprügeln, damit ein solche Behauptung die heilige Kuh der Expansion des Universums nicht in Frage stellen und damit am Urknalldogma kratzen kann …..

  24. #24 Florian Freistetter
    29. Juli 2013

    @dr feelgood: “So, jetzt dürft Ihr auf mich einprügeln, damit ein solche Behauptung die heilige Kuh der Expansion des Universums nicht in Frage stellen und damit am Urknalldogma kratzen kann …..”

    Bist du extra hier hergekommen, um den Märtyrer spielen zu können? Oder interessiert dich das Thema tatsächlich? Wenn ja, dann sollte dir bekannt sein, dass alternative Hypothesen zum Urknall seit Jahrzehnten ein wichtiger Teil der kosmologischen Forschung sind und von “Urknalldogma” keine Rede sein kann…

    Ansonsten: Wie GENAU soll denn der Zerfall von Photonen für die Rotverschiebung verantwortlich sein? Der Zerfall sorgt dafür, dass Photonen verschwinden. Bei der Rotverschiebung gehts um Photonen, deren Energie sich ändert.

  25. #25 Florian Freistetter
    29. Juli 2013

    @maunzz: In was sie zerfallen hängt von der Masse ab – die kennt man aber nicht. Zwei Neutrinos wären möglich. Neutralinos sind bis jetzt leider immer noch hypothetisch…

  26. #26 Alderamin
    29. Juli 2013

    @Dr. Feelgood

    Ganz ohne “drüber herfallen”: Das ist ja zunächst mal für sich gesehen gar kein schlechter Gedanke. Es gibt auch andere Argumente dafür, dass die Rotverschiebung des Lichts mit einer Expansion des Alls zusammen hängt, z.B. die Zeitdilatation von Supernovae-Ereignissen, deren Dauer wohlbekannt ist (insbesondere beim Type Ia) oder Gamma-Ray-Bursts. Die sind nämlich um den gleichen Faktor verlangsamt, um den die Wellenlänge des Lichts zunimmt (weil die selbe Ursache dahinter liegt).

    Neben der Rotverschiebung gibt’s aber auch genug andere, unabhängige Argumente für den Urknall, von der kosmischen Hintergrundstrahlung über das Verhältnis der Elemente in primordialem Gas bis zu der Entwicklung von Galaxien (Sternentwicklung, Wachstum, Aktivitätsphasen). Und schon der gute alte Einstein wusste lange vor Edwin Hubble, dass das Weltall gemäß seinen eigenen Formeln entweder kollabieren oder expandieren muss, und er schummelte voreilig an den Formeln herum, damit es das nicht tat, weil es damals unerhört war, dies anzunehmen. Was er später seine größte Eselei nannte.

  27. #27 Kallewirsch
    29. Juli 2013

    könnte der hier angesprochene Zerfall im Lauf von Jahrmilliarden nicht der wahre Grund für die beobachtete Rotverschiebeung im Lichtspektrum von sehr weit entfernten Objekten sein?

    Ich seh nicht, wie ein derartiger Zerfall die Rotverschiebung erklären könnte.
    Bei der Rotverschiebung gehts ja darum, dass jede Atomsorte ganz charakteristische Frequenzen aus dem Spektrum ausfiltert, was für dunkle Linien im Spektrum sorgt. Rotverschiebung wird gemessen, indem man sich ansieht, bei welcher Frequenz die Linie tatsächlich auftritt und das mit der Frequenz vergleicht, bei der sie auftreten sollte.
    Wenn Photonen zerfallen, dann sind sie weg. Aber davon verschieben sich keine Linien.

  28. #28 Rüdiger Kuhnke
    München
    29. Juli 2013

    Kallewitsch, lies das mit der Rotverschiebung nochmal nach, insbesondere wie sie zustandekommt.

  29. #29 Florian Freistetter
    29. Juli 2013

    @Rüdiger: “Kallewitsch, lies das mit der Rotverschiebung nochmal nach, insbesondere wie sie zustandekommt.”

    Stimmt schon, Kallewirsch erklärt wie die Spektrallinien entstehen, nicht wie sie verschoben werden. Trotzdem sehe ich nicht, wie der Photonezerfall eine Verschiebung der Spektrallinien simulieren soll.

  30. #30 Kallewirsch
    29. Juli 2013

    Kallewitsch, lies das mit der Rotverschiebung nochmal nach, insbesondere wie sie zustandekommt.

    Was gefällt dir daran nicht?

    Rotverschiebung bedeutet nun mal nicht, dass das Licht irgendwie ‘röter’ wäre, weil die blauen Photonen irgendwie abhanden gekommen wären (zb. indem sie auf dem Weg zerfallen). Das Spektrum als ganzes ist verschoben. Und das kann man daran erkennen, weil ein Spektrum eben nicht ein undifferenziertes etwas ist, sondern Charakteristika aufweist (eben die Linien), von denen man zweifelsfrei und eindeutig sagen kann, dass sie verschoben sind.

    insbesondere wie sie zustandekommt.

    Mir ging es in erster Linien nicht darum, wie sie zustande kommt, sondern darum aus welchem Argument man ableiten kann, wie sie ganz sicher nicht zustande kommt – zb dadurch, dass Photonen zerfallen.

  31. #31 Alderamin
    29. Juli 2013

    @Kallewirsch, Florian

    Na, wenn Photonen in Photonen geringerer Energie zerfielen, dann würden ja auch die Photonen von Emissionslinien zu längeren Wellenlängen verschoben. Und wenn die Photonen links und rechts einer Absorptionslinie in Photonen längerer Wellenlänge zerfielen, würde die Absorptionslinie ebenfalls verschoben.

    Aber ein Zerfall würde ja dann wohl gleich die Wellenlänge in einem Schlag verdoppeln, wenn sich die Energie symmetrisch auf zwei Photonen verteilte. Dann müsste man gequantelte Rotverschiebungen sehen. Sieht man aber nicht. Ist also Unsinn.

  32. #32 Chemiker
    29. Juli 2013

    @ Jarrus Poritschew

    Verzeiht einem Physiklaien..:
    Ich bin auch immer davon ausgegangen, dass Photonen, zumindest in Bewegung (& für stillstehende Photonen fällt mir kein Beispiel ein) eine Masse haben. Ich erinnere mich noch an das Experiment, wo ein Laser auf einen freihängenden Spiegel schoß und der Spiegel nach hinten schwang. Ohne Masse kein Rückstoß war meine Folgerung.

    Das ist leider nicht richtig; auch in der klassischen elektro­magnetischen Behandlung von Licht (Maxwell, 19. Jahrhundert, keine Licht­teilchen) bekommt man den Lichtimpuls richtig raus.

    MartinB hat das mal ausführlich erklärt, aber wahr­schein­lich mußt Du Dir dazu seine ganze Serie über die Maxwell-Gleichungen reinziehen.

    Und was ist mit E=mc²? Bei m=0 ist E auch 0…?

    Das mußt Du umgekehrt lesen: Die Energie eines Photons (hν = ħω) ist zu einer gewissen Masse äquivalent (Masse und Enegie sind ja dasselbe und über eine Konstante miteinander verbunden, die je nach gewählten Einheiten groß, klein oder auch Eins sein kann).

  33. #33 Alderamin
    29. Juli 2013

    Noch was:

    Wenn Photonen in andere Photonen zerfielen, hätte der Zerfall sicherlich (wie jeder andere auch) eine bestimmte Halbwertszeit, nach der erste die Hälfte zerfallen wäre, und auch nach sehr langer Zeit wären noch nicht alle Photonen einer bestimmten Quelle zerfallen. Damit würden die Linien verbreitert (oder verdoppelt, bei Quantelung). Was auch nicht der Fall ist.

  34. #34 Kallewirsch
    29. Juli 2013

    @Alderamin
    Die Idee ist ja nicht schlecht. Aber wie erklärt sich dann zb die Blauverschiebung, die wir bei Andromeda sehen. Das bei einem Zerfall 2 Teile rauskommen, die einzeln weniger Energie als das ursprüngliche Photon haben, kann ich ja noch akzeptieren, aber das da dann etwas rauskommt, was plötzlich mehr Energie hat?
    Und dann ist da ja auch noch die gravitative Rotverschiebung, die hier auf der Erde auf kurzen Distanzen gemessen wurde. Wenn die ebenfalls durch einen Zerfallsprozess zu erklären wäre, dann kann so ein Zerfall nicht besonders lange dauern. Was dann wieder das Problem aufwerfen würde: warum sehen wir dann überhaupt noch etwas aus weiterer Distanz.

    Ist also Unsinn.

    Die Idee an sich ist ja verfolgenswert. Aber wenn sogar ich (als Laie) innerhalb von 15 Minuten ein paar Problemkreise darin identifizieren kann, dann würde ich in stiller Selbstbescheidenheit sagen: So toll wird die Idee dann nicht gewesen sein 🙂

  35. #35 Rüdiger Kuhnke
    29. Juli 2013

    “Was gefällt dir daran nicht?”
    Daß es schlicht und einfach falsch ist.

  36. #36 Kallewirsch
    29. Juli 2013

    Daß es schlicht und einfach falsch ist.

    Was ist falsch?

    Dass die Idee eines Photonenzerfalls eine Verschiebung der Spektrallinien nicht erklären könnte?

    Inwiefern ist das falsch?

  37. #37 Rüdiger Kuhnke
    29. Juli 2013

    Ja. na, is klar. Ich geb’s auf.

  38. #38 Kallewirsch
    29. Juli 2013

    Dass die Idee eines Photonenzerfalls eine Verschiebung der Spektrallinien nicht erklären könnte?

    Vielleicht reden wir ja auch aneinander vorbei, oder ich hab mich schlecht ausgedrückt.
    Die Frage ist nicht:
    Wie entsteht die Rotverschiebung.
    Die Frage, um die es Feelgood ging lautete: Kann ein Photonenzerfall die Rotverschiebung erklären?

    Und IMHO: nein, das kann er nicht. Die Verschiebung der Spektrallinien zeigen, dass das nicht sein kann.

  39. #39 Alderamin
    29. Juli 2013

    @Kallewirsch

    Aber wie erklärt sich dann zb die Blauverschiebung, die wir bei Andromeda sehen.

    Indem es zwei Effekte gibt: den echten Dopplershift durch die Eigenbewegung und den kosmologischen durch den Zerfall der Photonen. Aber wie gesagt, ich hab’ ja selbst schon ein paar Argumente gebracht, dass das so nicht funktionieren würde.

    Nette Frage, schön, mal darüber nachgedacht zu haben. Nächste Frage?

  40. #40 johnny
    29. Juli 2013

    @Rüdiger Kuhnke
    Aufgeben setzt voraus, dass man es versucht hat.

  41. #41 Rüdiger Kuhnke
    29. Juli 2013

    Der Hinweis, daß er sowas von daneben liegt, war ja wohl genug Versuch. Renitentes Nichtdenken zu beheben, ist nicht meine Aufgabe. Claro?

  42. #42 Kallewirsch
    29. Juli 2013

    @Rüdiger

    Ganz ehrlich.
    Ich denke, dass du von etwas komplett anderem redest als ich. Ich geb mir wenigstens die Mühe, diesen Punkt erst mal zu klären. Von dir kommt nichts anderes als: falsch.

    Schönen Tag noch.

  43. #43 Florian Freistetter
    29. Juli 2013

    @Rüdiger Kuhnke: “Der Hinweis, daß er sowas von daneben liegt, war ja wohl genug Versuch. Renitentes Nichtdenken zu beheben, ist nicht meine Aufgabe. Claro?”

    Ich weiß jetzt nicht, wo auf einmal diese Aggression herkommt. Normalerweise ist “Falsch!!!” nicht unbedingt das, was man unter konstruktiver Kritik versteht. Wenn du tatsächlich an einer Diskussion interessiert bist, wäre es schon hilfreich, wenn du konkret sagen würdest, was an Kallewirsch Aussage falsch ist. Ansonsten weiß man nicht wirklich, worums dir geht. Gehörst zur Gruppe von Leuten, die der Meinung sind, Rotverschiebung die Expansion des Alls hätten nichts miteinander zu tun und der Urknall sei nur eine böse Lüge des Wissenschaftsdogmas, so wie Dr. Feelgood? Oder fandest du die Erklärung der Entstehung der Spektrallinien zu vereinfacht? Du hast nicht klar gesagt, was dich stört und auf die entsprechende (und eigentlich auch verständliche) Nachfrage gleich ein wenig böse und genervt reagiert. Keine gute Grundlage für ein vernünftiges Gespräch…

  44. #44 Alderamin
    29. Juli 2013

    Hmm, wenn man seine Posts bei Scienceblogs durchliest, ist er sonst auf der Seite der Wissenschaft. Vermutlich besteht hier, wie Kallewirsch schon anmerkte, ein massives Missverständnis.

    @Rüdiger Kuhnke

    Daß es schlicht und einfach falsch ist.

    Keiner von uns weiß, was genau Du mit “es” meinst.

  45. #45 Dietmar
    29. Juli 2013

    @Ferraristo, Dietmar:…Richtig

    Hah!

    Schön!

    😀

  46. #46 Niels
    29. Juli 2013

    @Bullet

    hmmm … allerdings frage ich mich, was denn die Bruchstücke sind, in die Photonen zerfallen können. Noch dazu, wo Photonen ja Bosonen mit dem Spin 1 sind und daher …

    @Silava

    Ganz einfach: Ein Teilchen ohne Masse ist immer mit Lichtgeschwindigkeit unterwegs. Und wer sich mit Lichtgeschwindigkeit bewegt, für den vergeht die Zeit nicht. Ein masseloses Teilchen hat also gar keine Zeit um zu zerfallen.

    @maunzz

    “Können Photonen zerfallen?”
    Und in was sollen sie zerfallen ? In 2 Neutralinos oder was ?

    Na ja, masselose Teilchen können durchaus “zerfallen” und das wird auch dauernd beobachtet.
    Das nennt man dann allerdings üblicherweise “Erzeugung” statt Zerfall. Ganz exakt heißt das bei Photonen Paarbildung oder Paarerzeugung, weil nur Teilchen-Antiteilchen-Paare entstehen können. Das ist einfach die Umkehrung des Annihilation-Prozesses (Paarvernichtung), bei dem sich Teilchen und zugehöriges Anti-Teilchen gegenseitig zu Photonen zerstrahlen.
    https://de.wikipedia.org/wiki/Paarbildung_(Physik)
    Über E=mc^2 steckt in der Energie eines Photons eben immer auch die Möglichkeit, Masse zu erzeugen.

    Für ein einzelnes isoliertes Photon ist das allerdings nicht möglich (zumindest wenn man von Masselosigkeit ausgeht).
    Ohne einen weiteren Partner, auf den Impuls übertragen werden kann, würde bei der Paarbildung nämlich entweder die Energieerhaltung oder die Impulserhaltung verletzt.

    Der übliche Partner bei solchen Vorgängen ist ein Atomkern oder ein Elektron, ein anderes Photon ist aber auch schon ausreichend.
    https://en.wikipedia.org/wiki/Matter_creation
    Kurz nach dem Urknall ist so etwas andauernd passiert.

    Es gibt sogar ein eigenes Teilgebiet der Teilchenphysik, bei dem man sich ausschließlich mit Photon-Photon-Wechselwirkungen beschäftigt.
    https://en.wikipedia.org/wiki/Two-photon_physics

    Das ist natürlich etwas völlig anderes als ein möglicherweise instabiles Photon mit einer Halbwertszeit, um das es bei dieser Forschungsarbeit geht.

    Es ist aber völlig normal ist, wenn sich Photonen in andere Teilchen umwandeln. Das ist nichts Überraschendes oder Zweifelhaftes, sondern ein bekannter, alltäglicher Vorgang. Auch wenn sie Bosonen mit Spin 1 sind und auch wenn “für sie keine Zeit vergeht”.
    (Übrigens ist Letzeres nicht ganz richtig. Tatsächlich kann man für masselose Teilchen nämlich gar keine Eigenzeit definieren. Das ist eine ganz andere Aussage als die “Eigenzeit ist Null”. Für ein (masseloses) Photon gibt es aber schlicht kein Inertialsystem.)

  47. #47 Luk
    29. Juli 2013

    https://www.pro-physik.de/details/news/5017871/Wie_lange_leben_Photonen.html

    hir gibts weitere infos, zur masse, zu den teilchen in die sie zerfallen könnten, lebensdauer, messgrafik etc.

  48. #48 griesl
    29. Juli 2013

    ich weiss nicht ob es passt, aber mir fällt da ein artikel aus der spektrum ein, in dem dargestellt wurde, dass es gelang aus einem quantenvakuum reale photonen aus virtuellen zu “erschaffen” ?!
    wenn photonen aus dem “nichts” entstehen können. können dann photonen nicht wieder ins quantenvakuum “verschwinden” ?

  49. #49 D.P.
    29. Juli 2013

    Ich glaube schon dasPhotonen eine Masse haben wenn auch nur eine sehr kleine.
    Sie werden ja von Gravitation angezogen. (Lichtbiegungen um Sterne usw.)

  50. #50 Alderamin
    29. Juli 2013

    @D.P.

    Nein, siehe #8 oben.

  51. #51 mr_mad_man
    29. Juli 2013

    Mal angenommen, man könnte (oder würde in der Zukunft) nachweisen, dass Licht tatsächlich Masse hat, welche Konsequenzen hätte das? Ich frage deshalb, weil es wahrscheinlich mehr wäre als die reine Feststellung “ok, Licht ist anders als wir bisher dachten”. Ich erinner mich noch an den Artikel über Gravitationswellen, das war es so, dass -wenn man in der Lage ist sie nachzuweisen- eine komplett neue Beobachtungsmethode für das Weltall offenstünde – was mir bis dahin überhaupt nicht klar war. Deshalb meine Frage, was würde sich durch einen Licht-hat-Masse-Nachweis ändern? Oder anders, könnte man irgendeinen praktischen Nutzen daraus ziehen?

  52. #52 Frank Melle
    30. Juli 2013

    Interessant. Vllt kann mir jmd eine Frage dazu beantworten?
    Angenommen, Photonen zerfallen und haben eine Masse. Welche Konsequenzen hätte das?

    Ich versuche grade zu verstehen, inwiefern man mit dieser Entdeckung bestimmte Dinge (welche?) besser erklären könnte.

    Ist sehr allgemein, aber vllt kann man das eine oder andere Bsp nennen.

  53. #53 Frank Melle
    30. Juli 2013

    Manchmal sollte man alle Beiträge lesen, auch wenn man ganz aufgeregt ist und unbedingt eine Frage stellen möchte…

  54. #54 Stephan
    30. Juli 2013

    Ich glaube, mal irgendwo gelesen zu haben, dass auch die “stabilen” Elementarteilchen, also Atomkerne (Up- und Down-Quarks) sowie Elektronen, nach x-Jahren (nach sehr langer Zeit, aber < als unendlich) "zerfallen" können (also instabil sind). Jetzt weiss ich aber nicht mehr, ob dies die Gluonen betraf (für mich als Laie sowas wie der Leim, welcher die Atomkerne, also die Quarks, zusammenhält) oder die Masseteilchen selbst. Weiss hier jemand von euch mehr oder hat jemand die Zeit, meine Frage zu beantworten? Lässt die "Existenz" (Kraft?) der Gluonen nach (danach schweben die Quarks einfach ohne Heimat im interstellaren Raum – haben Gluonen Kraft oder sind sie die Kraft selbst?) oder sind auch stabile Masseteilchen selbst eben nicht wirklich unendlich beständig (aber in Nichts können sie sich ja nicht auflösen)? Oder stimmt die ganze Annahme aus heutiger Sicht nicht (mehr)?

  55. #55 Alderamin
    30. Juli 2013

    @Stephan

    Meinst Du vielleicht den Protonenzerfall? Von einem Quark- oder Elektronenzerfall habe ich noch nie gehört, das sind ja Elementarteilchen. Protonenzerfall wurde bisher noch nie beobachtet.

    Die Kraft würde dabei nicht nachlassen, es würde sich eher um quantenmechanisches Tunneln handeln, vergleichbar dem radioaktiven Zerfall eines Kerns, denke ich, aber die Physiker hier werden das besser erklären können.

  56. #56 Stephan
    30. Juli 2013

    Nachtrag: Wenn man sich die Frage nach der Beständigkeit von Photonen stellt, könnte man sich diese Frage auch in Bezug auf die Gluonen stellen…? Vielleicht haben Gluonen doch eine Masse…? … und könnten “zerfallen”…?

  57. #57 Stephan
    30. Juli 2013

    @Alderamin: Jaaa, ich glaube, ich meinte den Protonenzerfall. Danke für den Hinweis. Nun, diese Halbwertszeit würde ja die Definition “Unendlichkeit” aushebeln: “Es wird eine Halbwertzeit von 10^31 bis 10^36 Jahren von den Theorien vorhergesagt.” (Quelle Wikipedia)

  58. #58 Alderamin
    30. Juli 2013

    @Stephan

    Da steht aber auch, dass nur einige der GUT-Theorien diesen Zerfall postulieren, und welche davon (falls überhaupt) korrekt ist, weiß man noch nicht. Insofern ist unendliche Lebensdauer durchaus noch drin.

  59. #59 H.M.Voynich
    30. Juli 2013

    “Vielleicht zerfallen die in DUNKLES LICHT!!! mit dem man dann die dunkle Materie sehen kann?”

    Hihi, dann hätte Terry Pratchett tatsächlich recht mit seiner “Dunkelgeschwindigkeit”, die höher ist als die Lichtgeschwindigkeit (da, egal wo das Licht hinkommt, die Dunkelheit schon vorher da war).

  60. #60 Lisa
    30. Januar 2014

    @ Alle
    Die Wellenansicht von gezeichneten Photonen zeigen ja abwechselnd ansteigende und absteigende Wellen.
    Doch dies nur gem. Grafik von der Seite betrachtet. Angenommen man bastelt sich zur Veranschaulichung so eine Photonenwelle und legt die Welle “flach auf die Seite”, so dass der Teiel des Drahtgebildes, der unten war, jetzt nach dem Hinlegen des Drahtes, man nur die “Untenansicht” sehen wurde, also kein Wellenmuster mehr. Auch die “Obenansicht”. Und wenn die Welle aus Draht zur anderen Seite auf den Tisch gelegt werden würde, und man von der Seite drauf schaut, dann sehe man das Gebilde nur von oben.
    Also wäre absolut keine Wellenmuster mehr zu sehen. Auch nicht aus der Vorderansicht und nicht aus der Hinteransicht. Allein nur von der Seite beobachtet.

    Auf was ich hinaus will? Das Universum müsste doch voller Photonenstrahlen sein, da das Licht von allen Seiten in alle erdenklichen Richtungen unterwegs ist. Es müsste überall aufeinander knallen, der ganze Raum müsste aus einem einzigen Photonenknäuel bestehen, demnach.
    Ist das eher Unsinn, wenn ich es mal so zu sehen versuche?

  61. #61 Hans-Werner
    30. Januar 2014

    Bleiben am Ende oder kurz vor dem Ende unseres Universums gemäß jetziger wissenschaftlicher Einschätzung nur noch Photonen übrig?
    Oder: Protonen, Elektronen, Neutrinos und Photonen?
    Beides habe ich vor Kurzem gelesen. Was ist richtig davon?

  62. #62 Florian Freistetter
    30. Januar 2014

    Lisa: “Ist das eher Unsinn, wenn ich es mal so zu sehen versuche?”

    Naja, nur weil man Photonen manchmal als Wellen zeichnet, darf man das nicht exakt auf die Realität übertragen. Photonen sind angeregte Bereiche des elektromagnetischen Feldes. Es ist aus praktischer Sicht manchmal einfacher, sie sich als Welle vorzustellen und dann kann es wieder praktischer sein, sie sich als Teilchen vorzustellen. Aber man darf es auch nicht übertreiben. Photonen können auch nicht im eigentlichen Sinne kollidieren. Du kannst im Prinzip beliebig viele Photonen in einen Bereich stecken; das stört die nicht…

  63. #63 Conz
    30. Januar 2014

    @Lisa
    Photonen können nicht kollidieren. Darum können ja mehrere Lichter (Laser) auf einen Punkt fokussiert werden, es wird dann nur die Energie höher. Es kann höchstens Interferenzen zwischen den Wellen geben.
    Auch ist Deine Vorstellung von den Wellen nicht ganz korrekt, Die Lichtwellen schwingen erst mal in alle Richtungen (quer zur Laufrichtung). Licht kann durch filter auch so gefiltert werden, dass als Ergebnis Lichtwellen raus kommen, die alle nur in einer Ebene schwingen, das ist dann polarisiertes Licht.
    https://de.wikipedia.org/wiki/Polarisiertes_Licht

  64. #64 Hans-Werner
    30. Januar 2014

    https://www.thur.de/philo/tanja/expansion/11.htm
    Verstehe ich das richtig, dass gemäß dieser Seite nach dem Urknall als erstes die Photonen da waren?

  65. #65 Florian Freistetter
    30. Januar 2014

    @Hans-Werner: Ich weiß nicht, was das für eine Seite ist (das letzte Mal aktualisiert wurde sie jedenfalls im Jahr 2002). Wenn du den Urknall wirklich überblicken willst, empfehle ich dir dieses Buch: https://scienceblogs.de/astrodicticum-simplex/2009/01/27/big-bang-und-big-bang-zweiter-akt-simon-singh-vs-harald-lesch-teil-1/

    Photonen sind ja Energie und natürlich war die als erstes da. Aus der Energie sind jede Menge Elementarteilchen und jede Menge Photonen entstanden. Und später daraus dann die normale Materie.

  66. #66 Lisa
    2. Februar 2014

    @Conz und @Florian Freistetter,
    “Deine Vorstellung von den Wellen nicht ganz korrekt, Die Lichtwellen schwingen erst mal in alle Richtungen (quer zur Laufrichtung).”
    Danke Euch beiden für die schnelle Antworten. Gut, dass sie in alle Richtungen schwingen, zog ich zwar auch in Erwägung, hätte es aber nicht gedacht.
    Worin ich nicht mit euch einig gehe, ist, dass Photonen nicht miteinander kollidieren können. Das mag auf die Aktivität im beobachtbaren Universum stimmen, doch der Teilchenbeschleuniger in Stanford (Kalifornien) hat ja Photonen aufeinander zuknallen lassen und somit Protonen erzeugt.
    Zitat
    “In Stanford dagegen wurden reale Lichtblitze aufeinandergeschossen. Mittels einer trickreichen Versuchsanordnung pumpten die Physiker dabei für eine billionstel Sekunde so viel Energie in einen milliardstel Quadratzentimeter, wie die gesamten Vereinigten Staaten in derselben Zeit verbrauchen. Eine der stärksten Laserquellen der Welt sandte extrem kurze Lichtpulse aus. Um einen zweiten, noch energiereicheren Strahl zu erzeugen, lenkten die Physiker dieses Laserlicht genau auf den feingebündelten Elektronenstrahl, der im Stanforder Beschleuniger erzeugt wird. Wie Pingpongbälle auf einen Lastwagen prallten die Lichtteilchen auf die Elektronen und wurden mit noch mächtigerem Stoß zurückgeworfen. Aus sichtbarem grünem Laserlicht wurde so höherenergetische Gammastrahlung, die mit dem nächsten Laserpuls kollidierte.” Zitatende
    Auszug aus Seite 1 von 2 Seiten.
    Quelle: https://www.zeit.de/1997/42/materie.txt.19971010.xml

  67. #67 Florian Freistetter
    2. Februar 2014

    @Lisa: ” Wie Pingpongbälle auf einen Lastwagen prallten die Lichtteilchen auf die Elektronen”

    Da steht es doch… Photonen kollidieren mit ELEKTRONEN. Aber nicht Photonen mit Photonen…

  68. #68 Niels
    2. Februar 2014

    @Florian
    Lisa bezieht sich auf diesen Teil:

    Aus sichtbarem grünem Laserlicht wurde so höherenergetische Gammastrahlung, die mit dem nächsten Laserpuls kollidierte.

    Das ist auch tatsächlich so gemeint. An anderen Stellen im Artikel:

    Tatsächlich gleicht also das Vakuum in Wahrheit einem See, in dem unablässig Energie und Materie fluktuieren. Im Mittel addieren sich diese Fluktuationen stets zu null. Wenn allerdings in einem derart brodelnden Nichts zwei Lichtblitze mit Wucht zusammenprallen, dann kann diese Energie durchaus einem gerade fluktuierenden Partikelpaar zum Sprung von der möglichen in die reale Existenz verhelfen.

    Die zusammenprallenden Lichtstrahlen erzeugten lediglich ein “kritisches” elektromagnetisches Feld, das die Entstehung je eines Elektron-Positron-Paares aus dem puren Nichts beförderte.

    Mir ist überhaupt nicht klar, worum es in diesem Artikel überhaupt geht. Dort steht sowieso ziemlich viel Unsinn, der Autor hat wenig Ahnung von dem, worüber er schreibt.

    In Stanford gibt es doch nur das SLAC National Accelerator Laboratory. Dort ließ man Elektronen und Positronen kollidieren.
    Der Artikel ist zwar ziemlich schlecht, aber ob der Autor wirklich nicht einmal das verstanden hat? Oder was könnte sonst gemeint sein?

  69. #69 Florian Freistetter
    2. Februar 2014

    @Niels: Ich vermute mal, da geht es um das hier: https://en.wikipedia.org/wiki/Two-photon_physics

    Da geht es um eine Art “Streuung” von hochenergetischen Photonen aneinander. Da gehts um Vakuumfluktuationen die von Photonen angeregt werden können die wiederum andere Photonen beeinflussen. Aber eine echte “Kollision” ist das nicht…

  70. #70 Lisa
    3. Februar 2014

    Danke Niels.
    Dass der einen Haufen Unsinn schreibt, daran hatte ich bis jetzt nicht gedacht. Doch ziehe es ab jetzt zumindest in Erwägung.
    Mal schauen, ob ich heraus finden kann, inwieweit die Kernaussage, um die es mir geht, von der Wissenschaft als seriöse eingeschätzt wird oder nicht:
    Dass gemäß dem aufwendigen Experiment aus 1997, auch wenn es offensichtlich kaum in den Medien Erwähnung fand, Photonen mit Photonen zusammen knallten bzw. “prallten” mit diesem phänomenalen Ergebnis, dass nämlich aus Nichtmaterie Materie gebildet wurde, was im letzten Teil des folgenden Zitats deutlich zum Ausdruck kommt.
    Zitat: “Nach insgesamt rund vierjähriger Arbeit hatten sie gerade einmal rund einhundert Elektronen – sowie deren Antiteilchen, die Positronen – erzeugt. Mit dieser Ausbeute, für die in Stanford mehrere Billionen Watt verheizt wurden, ließe sich nicht einmal ein Taschenlämpchen zum Glühen bringen. Eine verheerende Energiebilanz. Aber es geht ja auch mehr ums Prinzipielle: Aus reiner Energie läßt sich Materie verstofflichen. Welch eine Symbolkraft steckt in diesem Akt.”

  71. #71 Lisa
    3. Februar 2014

    Sorry, ich meinte den vorletzten Satz, nicht den letzten 🙂

  72. #72 Lisa
    3. Februar 2014

    @Florian
    Du schreibst oben u. a. diese interessante von Julian Heeck geäußerte Aussage: >>Die Theorie erlaubt also, dass Licht eine Masse hat und wenn das theoretisch möglich ist, dann gibt es keinen Grund, warum man einfach so von masselosen Photonen ausgehen sollte”.

    Meine Frage: Hat das, was ich in diesem Artikel fand, etwas damit zu tun?
    https://www.golem.de/news/neuer-materiezustand-lichtteilchen-bilden-molekuele-1309-101819.html

    U. a. heißt es dort, Zitat:
    “US-Wissenschaftler haben Photonen dazu gebracht, so miteinander zu interagieren, dass sie Moleküle bilden. Das sei ein neuer Materiezustand, sagen die Forscher.”
    ———-
    Zu dem von dir angeführten Link
    https://en.wikipedia.org/wiki/Two-photon_physics
    Ich glaube fast nicht, dass dort das beschrieben steht, was in Stanford angeblich erreicht wurde.

    Selbst auf dieser relativ neuen Seite:
    https://translate.googleusercontent.com/translate_c?depth=1&hl=de&prev=/search%3Fq%3Dtwo-photon%2Bphysics%26biw%3D1280%26bih%3D878&rurl=translate.google.de&sl=en&u=https://physics.stackexchange.com/questions/54323/collision-of-two-photons&usg=ALkJrhiMSsvsU9pTZdCHz7HZ0H_qM3fN5Q

    Ist man unter Fachleuten ähnlicher Ansicht wie du. Doch ich habe über das Kontaktformular das Folgende hin geschrieben (weil eine Registrierung bis jetzt nicht möglich war):

    Das hier wollte ich vorhin also veröffentlichen und stelle die gleichen unter dem Text gestellten Fragen eigentlich auch an Euch.

    @Frederic Brünner
    Was ich hier unter den beiden Links gefunden habe, zeigt, dass Ihre Annahme falsch sein dürfte, wenn Sie schreiben, dass zwei Photonen nicht miteinander kollidieren oder interagieren können.

    1. Link
    Quelle https://www.golem.de/news/neuer-materiezustand-lichtteilchen-bilden-molekuele-1309-101819.html
    Zitat /Auszug: “US-Wissenschaftler haben Photonen dazu gebracht, so miteinander zu interagieren, dass sie Moleküle bilden. Das sei ein neuer Materiezustand, sagen die Forscher.”
    2. Link
    Quelle https://www.zeit.de/1997/42/materie.txt.19971010.xml
    Zitat / Auszug 1/2
    “Amerikanischen Physikern ist nun ein solches Experiment gelungen. Zwanzig Wissenschaftler von vier amerikanischen Universitäten wandelten am Linearbeschleuniger in Stanford, Kalifornien, erstmals pures Licht in Materie um,”
    Zitat / Auszug 2/2
    “Nach insgesamt rund vierjähriger Arbeit hatten sie gerade einmal rund einhundert Elektronen – sowie deren Antiteilchen, die Positronen – erzeugt. Mit dieser Ausbeute, für die in Stanford mehrere Billionen Watt verheizt wurden, ließe sich nicht einmal ein Taschenlämpchen zum Glühen bringen. Eine verheerende Energiebilanz. Aber es geht ja auch mehr ums Prinzipielle: Aus reiner Energie läßt sich Materie verstofflichen. Welch eine Symbolkraft steckt in diesem Akt.”

    Falls beide Artikel-Aussagen korrekt sind, finde ich es schade, dass so wenige Medienberichte darüber existieren und schade, dass viele Wissenschaftler sogar davon (noch) nichts wissen.
    ————
    Meine zwei hauptsächlichen Frage sind: Stimmt das, was die Artikel aussagen? Wer hat Verbindung zu Linearbeschleuniger in Stanford, Kalifornien, um dort persönlich nachzufragen, ob das im 2. Link Behauptete der Wahrheit entspricht?

  73. #73 Lisa
    3. Februar 2014

    @Florian,
    Doch ich sehe mir gerade einen interessanten Austausch in diesem Forum an…
    https://www.quanten.de/forum/archive/index.php5?t-2141.html
    Da ist ebenfalls von der Sache mit Stanford die Rede.
    Ein Hawkind gibt dir aber quasi dort Recht in seinem Kommentar um 12:02 Uhr, soweit ich das beurteilen kann, indem er ebenfalls schreibt, dass nicht Photoen auf Photonen geschossen wurden, sondern Photonen auf Elektronen.

    Was stimmt denn jetztz?? Wer vertritt hier jeweils lediglich eine Ansicht, weil dieser Artikel über Stanford undeutlich formuliert ist und wer weiß oder kriegt raus, was wirklich Sache ist? Denn ich denke gleichzeitig daran, was Niels schrieb bzw. zitierte, dass “zwei Lichtblize mit Wucht zusammen prallten”.

    Doch mit “Lichtblitze können ja ebenfalls Photon und Elektronen von diesem Autor gemeint gewesen sein. Somit müsste ich das bisher von mir Missverstandene korrigieren.

  74. #74 Lisa
    3. Februar 2014

    So, nun nach weiterem Forschen und Surfen, denke ich, habe ich es differenzieren können und komme, bis mich jemand gegen überzeugt 🙂 , wieder zu meiner ersten Überzeugung zurück duch diesen Abschnitt, den ich – zusammen mit dem, was Niels anführte -, erst jetzt meine, verstanden zu haben, und in dem ich jeweils 3 Ausrufezeiten vor und hinter den wesentlichen Begriff setze. Denn wie man fett formatiert oder kursiv, weiß ich leider nicht.
    Hier zuerst der Abschnitt im Ganzen:
    >>In Stanford dagegen wurden reale Lichtblitze aufeinandergeschossen. Mittels einer trickreichen Versuchsanordnung pumpten die Physiker dabei für eine billionstel Sekunde so viel Energie in einen milliardstel Quadratzentimeter, wie die gesamten Vereinigten Staaten in derselben Zeit verbrauchen. Eine der stärksten Laserquellen der Welt sandte extrem kurze Lichtpulse aus.<<
    ———
    Und hier, was ich meine:
    Um einen !!!zweiten!!!, noch energiereicheren Strahl zu !!!erzeugen!!!, lenkten die Physiker !!!dieses Laserlicht!!! genau auf den feingebündelten Elektronenstrahl, !!!der im Stanforder Beschleuniger !!!erzeugt!!! wird (nämlich erzeugt von diesem ersten Strahl!!! (welcher als Ergebnis des Photonen-Photonen-Beschusses ERZEUGT wurde).
    Wie Pingpongbälle auf einen Lastwagen prallten die Lichtteilchen auf die Elektronen (welche, um es noch einmal zu erwähnen, !!!allerdings ja gerade erst!!! durch das Aufeinanderprallen der Photonen mit Photonen !!!erzeugt!!! wurden) und wurden mit noch mächtigerem Stoß zurückgeworfen. Aus sichtbarem grünem Laserlicht wurde so höherenergetische Gammastrahlung, die mit dem nächsten Laserpuls kollidierte.

  75. #75 Florian Freistetter
    3. Februar 2014

    @Lisa: “Meine Frage: Hat das, was ich in diesem Artikel fand, etwas damit zu tun?
    https://www.golem.de/news/neuer-materiezustand-lichtteilchen-bilden-molekuele-1309-101819.html

    Nein, gar nicht. Das mit den Photonen und der Masse ist eine höchst hypothetische Spekulation; einfach nur ein “Was wäre wenn”-Gedankenspiel, das theoretische Physiker gerne treiben. Das was bei Golem beschrieben wurde beschreibt ein konkretes Experiment das weder belegt das Photonen eine Masse haben noch sonst irgendwie mit dem Artikel hier in Verbindung steht.

  76. #76 Florian Freistetter
    3. Februar 2014

    @Lisa: Ich kann dir nicht sagen, was jetzt genau in Stanford gemacht wurde weil ich dort nicht dabei war. Aber wie im verlinkten Wikipedia-Artikel beschrieben handelt es sich trotzdem nicht um “echte” Kollisionen. Es stoßen nicht zwei Lichtteilchen im klassischen Sinn zusammen und es macht “Peng” und irgendwas neues entsteht. Da geht es um komplizierte quantenmechanische Anregungsprozesse zwischen Lichtteilchen.

  77. #77 Alderamin
    3. Februar 2014

    @Niels

    Weiß nicht, ob wir das schon einmal diskutiert hatten, aber wie ist eigentlich die erste Materie aus der Strahlung nach dem Urknall entstanden? Lässt sich der Prozess der Zerstrahlung eines Positrons und eines Elektrons nicht auch zeitlich umkehren?

    Die so entstandenen Leptonen könnten dann die Keimzelle für die Erzeugung von Baryonen gewesen sein. Oder wie stellt man sich das vor?

  78. #78 JaJoHa
    3. Februar 2014

    Photonenmasse<10^{-18}eV ist ein aktueller Wert für die maximale Photonenmasse (obere Grenzem laut Particle Data Group.

  79. #79 JaJoHa
    3. Februar 2014

    @Alderamin
    Annihilation läuft über elektromagnetische WW, sollte also unter Zeitumkehr genauso laufen (da die Wechselwirkung CP und CPT erhält muss T auch erhalten werden und der Prozess geht in beide Richtungen).
    Da müssen auch keine Leptonen stehen (und Neutrinos kannst du so übrigens nicht erzeugen, da sie keine Ladung haben, da müsste dann ein Z-Boson stehen), da könntest du auch beliebige Quarks direkt mit erzeugen. Hier kannst du dir die erlaubten Kopplungen anschauen, du dürftest also ein (virtuelles) Photon auch in in Paar W-Bosonen gehen lassen und die in Hadronen.

  80. #80 JaJoHa
    3. Februar 2014

    @Lisa
    Falls du wissen willst wieso die Photonen durch die schnellen Elektronen energiereicher werden, das ist sehr Wahrscheinlich inverser Comptoneffekt.

  81. #81 Niels
    3. Februar 2014

    @Lisa

    Mal schauen, ob ich heraus finden kann, inwieweit die Kernaussage, um die es mir geht, von der Wissenschaft als seriöse eingeschätzt wird oder nicht:
    Dass gemäß dem aufwendigen Experiment aus 1997, auch wenn es offensichtlich kaum in den Medien Erwähnung fand, Photonen mit Photonen zusammen knallten bzw. “prallten” mit diesem phänomenalen Ergebnis, dass nämlich aus Nichtmaterie Materie gebildet wurde

    Photonen können tatsächlich so miteinander oder mit anderen Teilchen wechselwirken, dass aus Photonen Teilchen und Antiteilchen entstehen.
    Dazu habe ich weiter oben in diesem Thread schon einmal etwas geschrieben:
    https://scienceblogs.de/astrodicticum-simplex/2013/07/29/instabiles-licht-konnen-photonen-zerfallen/#comment-211203

    Ich verstehe nur nicht, was das mit Stanford und dem Jahr 1997 zu tun hat. Paarbildung, in diesem Fall die Erzeugung von Elektron-Positron-Paaren aus Photonen, wurde schon 1933 experimentell durchgeführt, 1948 gabs dafür sogar den Nobelpreis.
    Die Überschrift des Zeitartikels (Amerikanische Physiker schufen erstmals Materie aus reinem Licht) ergibt für mich deswegen keinen Sinn.

    Paarbildung ist übrigens ein ziemlich normaler Vorgang bei den heutigen Teilchenbeschleunigern. Beim Large Hadron Collider (LHC) kommt das andauernd als “Nebenprodukt” vor.

    @Alderamin

    Lässt sich der Prozess der Zerstrahlung eines Positrons und eines Elektrons nicht auch zeitlich umkehren?

    Richtig. Siehe oben. Zerstrahlung ist aber immer möglich, während man für die Erzeugung sehr energiereiche Photonen benötigt.

    Weiß nicht, ob wir das schon einmal diskutiert hatten, aber wie ist eigentlich die erste Materie aus der Strahlung nach dem Urknall entstanden?

    Ich bin mir ziemlich sicher, dass wir das schon einmal hatten und du es auch vollkommen richtig verstanden hast.

    Ganz kurz (und unzulässig vereinfacht 😉 ):
    Das Inflatonfeld “zerfällt” nach der Phase der Inflation in alle möglichen exotischen Teilchen-Antiteilchen-Paare, in Photonen und (über Zwischenschritte?) in dunkle Materie.
    Diese exotischen Teilchen-Antiteilchen-Paare zerstrahlen sich zu Photonen oder zerfallen in weniger exotische Teilchen, die sich wieder entweder zerstrahlen oder weiterzerfallen usw. Am Schluss bleibt wegen der Symmetriebrechung ein Rest Materie und ein Haufen Strahlung übrig. Das Universum kühlt sich aufgrund der Expansion irgendwann so weit ab, dass das Quark-Gluon-Plasma zu Protonen und Neutronen kondensiert. Dann folgt die primordiale Nukleosynthese, also die Fusionsreaktionen zwischen den Protonen und fertig ist die Laube.

    Da alles so dicht gepackt ist, findet dabei auch andauernd Paarbildung statt. Da entstehen natürlich nicht nur Positronen und Elektronen, sondern alle Teilchen-Paare, für die die Energie der Photonen ausreicht. Durch die Expansion des Universums werden die Photonen aber immer energieärmer, mit der Zeit fallen also immer mehr Paarbildungs-Möglichkeiten weg.

    Die so entstandenen Leptonen könnten dann die Keimzelle für die Erzeugung von Baryonen gewesen sein. Oder wie stellt man sich das vor?

    Ich verstehe die Frage nicht ganz. Die Leptogenese und die Baryogenese sind noch unverstanden. Es gilt aber als ziemlich sicher, dass beides irgendwie zusammenhängt. Da macht man dann mit Sphaleronen, Instantonen und so Zeug rum. Da kenne ich mich aber nicht wirklich aus.

  82. #82 Alderamin
    3. Februar 2014

    @Niels

    Das Inflatonfeld “zerfällt” nach der Phase der Inflation in alle möglichen exotischen Teilchen-Antiteilchen-Paare,

    Ach so, ich hatte gedacht, das Inflatonfeld setzte seine Energie zunächst nur als Photonen frei, die dann bei weiterer Abkühlung zu Teilchen-Antiteilchenpaaren kondensierten.

    Ich verstehe die Frage nicht ganz.

    War auch Unfug, ich hatte aus den Posts oben etwas missverstanden. Jedenfalls entsteht bei der Zerstrahlung von Protonen und Antiprotonen nicht nur Energie, sondern es wird auch ein Meson frei, weil die Baryonenzahl erhalten bleiben muss. Ein Erhaltungsgesetz, das bei der Erklärung der Baryogenese wohl das eigentliche Problem sein dürfte, korrekt?

  83. #83 Niels
    3. Februar 2014

    @Alderamin
    Na ja, wird reden über das Inflaton-Feld. Präziser gesprochen “zerfällt” das Inflaton-Feld in alle bekannten Quantenfelder, also beispielsweise auch ins Higgs-Feld. Bei Quantenfeldtheorien blicke ich aber nur gerade so bei den absoluten Grundlagen durch, sorry. Da kann dir MartinB aber wahrscheinlich weiterhelfen.
    Das ist aber natürlich alles nur theoretische Spekulation, noch lange kein gesichertes Wissen.

    Klar, die Baryonenzahlerhaltung muss irgendwie verletzt werden.
    Die Frage ist wie bzw. warum.

    Das kann man vor allem deswegen noch nicht beantworten, weil wir die drei quantenmechanischen Kräfte bisher noch nicht vereinheitlichen konnten. Das gelang bisher nur bei der schwachen und der elektromagentischen Kraft zur elektroschwachen Kraft. Die starke Kraft steht aber noch völlig allein.
    Gerade im frühen Universum sind die Energien aber so groß, dass man mit höchstwahrscheinlich mit getrennten Beschreibungen nicht weiterkommt.
    Man nimmt beispielsweise an, dass statt der Baryonenzahl etwas anderes, zum Beispiel die sogenannte schwache Hyperladung oder die Differenz von Baryonenzahl und Leptonenzahl erhalten bleibt bzw. blieb. Sogenannte Sphaleronen wandelten Baryonen in Anti-Leptonen und Anti-Baryonen in Leptonen um.
    Dann wären Leptonen tatsächlich irgendwie die “Keimzelle” für Baryonen gewesen.
    In some theories of baryogenesis an imbalance of the number of leptons and antileptons is formed first by leptogenesis and sphaleron transitions then convert this to an imbalance in the numbers of baryons and antibaryons.

    (Witzigerweise konzentriert man sich in der Forschung mittlerweile gleich auf die theory of everything (ToE) aller vier Grundkräfte, weil man bei dieser grand unified theory (GUT) der drei quantenmechanischen Grundkräfte nicht weiterkam.
    Nur steckt man jetzt bei der ToE meiner Meinung nach aber mindestens genau so schlimm in der Sackgasse.)

  84. #84 Alderamin
    3. Februar 2014

    @Niels

    Danke für die Erläuterung.

  85. #85 Lisa
    3. Februar 2014

    @Niels
    Danke für diese für mich wichtige Information von dir:
    >>Photonen können tatsächlich so miteinander oder mit anderen Teilchen wechselwirken, dass aus Photonen Teilchen und Antiteilchen entstehen.
    Dazu habe ich weiter oben in diesem Thread schon einmal etwas geschrieben:
    https://scienceblogs.de/astrodicticum-simplex/2013/07/29/instabiles-licht-konnen-photonen-zerfallen/#comment-211203

    Ich verstehe nur nicht, was das mit Stanford und dem Jahr 1997 zu tun hat. Paarbildung, in diesem Fall die Erzeugung von Elektron-Positron-Paaren aus Photonen, wurde schon 1933 experimentell durchgeführt, 1948 gabs dafür sogar den Nobelpreis.
    Die Überschrift des Zeitartikels (Amerikanische Physiker schufen erstmals Materie aus reinem Licht) ergibt für mich deswegen keinen Sinn.<>Mittlerweile ist so ein Prozess auch bebachtet worden: Photonen aus Laserquellen wurden zur Kollision gebracht … mit beobachteter Paarerzeugung:

    a collision between the high-energy photon and several laser photons to produce an electron-positron pair. These results are the rst laboratory evidence for inelastic light-by-light scattering involving only real photons. >>

    Ich nehme an, da handelt es sich um das Gleiche.

  86. #86 Lisa
    3. Februar 2014

    @Florian,
    bleibst du bei deinem Standpunkt, Photonen könnten mit Photonen nicht im klassischen Sinn kollidieren, obwohl es sich so verhält, wie Niels es beschreibt?

    Und wie Hawkwind schreibt::
    >>Mittlerweile ist so ein Prozess auch bebachtet worden: Photonen aus Laserquellen wurden zur Kollision gebracht … mit beobachteter Paarerzeugung<<

    Mein Fehler lag ja darin, dass ich meinte, in Stanford sei etwas Neues Sensationelles nachgewiesen worden.
    Dabei war es nichts Neues, wie Niels es mir erklärte.

    Warum ich dir aber hier die Frage stellte, ob du weiterhin dabei bleibst…, hat einfach den Grund:

    Da mir jetzt zwei verschiedene Aussagen vorliegen, nämlich die eine von Niels, die mir natürlich sehr gefällt, und die Andere von dir, in der du nach wie vor behauptest, Photonen könnten nicht (quasi definitiv im klassischen Stil) mit Photonen in Kollision gebracht werden,
    möchte ich weiterhin wissen, welche Aussage richtig ist gemäß neuestem Stand der Forschung.

    Oder sagst du etwa im Grunde das Gleiche wie Niels und wie Hawkwind und nur ich kapierte das bisher nicht? Ist nicht ironisch gemeint, ich könnte einfach auf dem Schlauch stehen. Doch bis jetzt nehme ich an, dass Niels und du nicht das Gleiche meinen.

  87. #87 Florian Freistetter
    3. Februar 2014

    @Lisa: “bleibst du bei deinem Standpunkt, Photonen könnten mit Photonen nicht im klassischen Sinn kollidieren, obwohl es sich so verhält, wie Niels es beschreibt?”

    Ja. Photonen sind keine physischen Objekte die mit einem “Peng” zusammenstoßen können. Und ich denke, ich meine nichts anderes als Nils. Photonen können miteinander wechselwirken so wie in diesem Wiki-Artikel beschrieben: https://en.wikipedia.org/wiki/Two-photon_physics. Das ist auch das, was in Stanford gemacht wurde. Aber das ist nicht das, was man i.A. unter dem Begriff “Kollision” versteht.

    Wenn man es ganz genau beschreiben will, macht es sowieso keinen Sinn, von den Photonen als Teilchen zu sprechen bzw. als Welle. Die moderne Quantenmechanik beschreibt alles nur noch in Form von Feldern, die angeregt werden können. Und wenn du genug Energie in ein Photonfeld steckst, dann kann dabei eben auch zB das Elektronfeld angeregt werden bis ein Elektron “rauskommt”. Aber die Quantenfeldtheorie lässt sich jetzt mal nicht eben so in nem Kommentar erklären (Martin hat nebenan in seinem Blog “Hier wohnen Drachen” sehr viel dazu geschrieben.

    “Da mir jetzt zwei verschiedene Aussagen vorliegen, nämlich die eine von Niels, die mir natürlich sehr gefällt,”

    Darf ich fragen, warum es dir so “sehr gefällt” wenn jemand sagt, dass Photonen kollidieren? Hast du da irgendein persönliches Interesse daran, dass sie das tun?

  88. #88 Lisa
    3. Februar 2014

    @Florian,
    dass es sich hier, wie du oben schreibst, um eine höchst hypothetische Spekulation handelt, kann zwar möglich sein, doch geht das nicht aus dem angeführten Artikel hervor, sondern im Gegenteil, das Experiment wurde real durchgeführt, so wie ich das verstehe.
    Beziehst du dich vielleicht auf andere von dir gelesene Artikel und hast diesen hier nicht ganz durch gelesen? Denn das passiert mir selbst leider andauernd. Viele Artikel lese ich ganz und viele überfliege ich nur, wobei ich natürlich da an mir arbeiten muss, doch daher behaupte ich fast nie etwas, weil ich irgendwo weiß, wie leicht ein Missverständnis beim Lesen oder Überliegen passieren kann.
    ———————–
    >>Ausgangspunkt des Experiments war eine Wolke von Rubidiumatomen in einer Vakuumkammer. Mit Hilfe eines Lasers kühlten die Wissenschaftler die Wolke bis auf wenige Grad über dem absoluten Nullpunkt ab. Als Nächstes schossen sie mit einem schwachen Laser ein einzelnes Photon durch die Wolke. Das Lichtteilchen gab seine Energie an die Atome ab. Die Energie wurde von Atom zu Atom weitergegeben und verließ am Ende zusammen mit dem Photon die Wolke wieder.
    Wie Lichtbrechung

    “Wenn das Photon das Medium verlässt, bleibt seine Identität erhalten”, sagt Lukin. Das sei ähnlich wie die Lichtbrechung im Wasser: “Das Licht kommt in das Wasser, gibt einen Teil seiner Energie an das Medium ab. Drinnen existiert es als Licht und Materie, die miteinander verbunden sind. Aber wenn es aus dem Wasser herauskommt, ist es immer noch Licht.” Nur gebe das Licht in der Wolke mehr Energie ab und werde stark gebremst.

    Als die Forscher aber zwei Photonen in die Rubidiumwolke schossen, passierte etwas Bemerkenswertes: Sie verließen die Wolke zusammen, als ein einziges Molekül. In der Wolke wirkt ein bestimmter Effekt: Wird ein Atom angeregt, können weitere Atome in dessen Nähe nicht in gleichem Maße angeregt werden. Wenn also zwei Photonen in die Wolke eindringen, regt das erste ein Atom an. Das zweite kann aber die Atome in der Nähe erst dann anregen, wenn sich das erste weiterbewegt hat.<<
    —————————
    Quelle: https://www.golem.de/news/neuer-materiezustand-lichtteilchen-bilden-molekuele-1309-101819.html

  89. #89 Florian Freistetter
    3. Februar 2014

    @Lisa: “Beziehst du dich vielleicht auf andere von dir gelesene Artikel und hast diesen hier nicht ganz durch gelesen?”

    Ich habe gemeint: MEIN Artikel über instabile Photonen hat NICHTS mit irgendwelchen Experimenten in Stanford und auch NICHTS mit dem Golemartikel über “Lichtmoleküle” zu tun. Der Golemartikel ist außerdem sehr schlecht geschrieben. Allein der Satz “Als die Forscher aber zwei Photonen in die Rubidiumwolke schossen, passierte etwas Bemerkenswertes: Sie verließen die Wolke zusammen, als ein einziges Molekül.” ist schon Unsinn. Ein Molekül ist ein Verbund einzelner Atome. Licht ist kein Atom. Licht ist keine Materie und kann keine Moleküle bilden. Das ist nur ein (schlechter) Vergleich für ein spezielles Phänomen das hier bei zwei Lichtteilchen/wellen gemessen wurde.

    Es wäre vielleicht hilfreich zu wissen, um was es dir eigentlich geht. Die Stanford-Sache, die “Lichtmoleküle” und mein Artikel sind drei ganz unabhängige Berichte über Forschung die halt alle drei zufällig mit Licht zu tun haben. Gibt es irgendeinen Grund warum das in Verbindung bringst oder willst du einfach nur mehr über Licht wissen? Dann wäre es gut, das nicht alles immer durcheinander zu schmeissen.

  90. #90 Lisa
    3. Februar 2014

    Bevor ich dir deine Frage beantworte, lass mich bitte noch einmal darauf zurück kommen, was hier geschrieben wurde:
    Positron production in multiphoton light-by-light scattering

    >>Mittlerweile ist so ein Prozess auch bebachtet worden: Photonen aus Laserquellen wurden zur Kollision gebracht … mit beobachteter Paarerzeugung<<

    und dich fragen, ob es sich hier nicht um Kollision handelt, wenn aus Lasern Photonen zur Kollision gebracht werden?

    Wie du mir jedesmal erklärst, dass dies keine Kollision sei.
    Und wenn du als Ergebnis Elektrionen und Positronen raus kommen.
    Natürlich kann so ein auf der Erde – und nicht im Universum – durchgeführter Versuch nur minimale Ergebnisse zeitigen. Doch es geht ums Prinzipielle, darum, dass wie auch Niels es anführt, durch Kollision von nur Photonen das entsteht, wie beschrieben.
    Auch sein (Niels) o.a. Beitrag schreibt davon.

    So denke ich noch immer, dass Niels und du nicht das gleiche meint.

  91. #91 Florian Freistetter
    3. Februar 2014

    @Lisa:

    1) Photonen aus zwei Lasern wechselwirken miteinander.
    2) Aus der Energie dieser Wechselwirkung entstehen neue Teilchen.
    3) Es handelt sich nicht um eine Kollision.

    Wenn du diese Art von Wechselwirkung, die im schon mehrmals erwähnten Wikipedia-Artikel (https://en.wikipedia.org/wiki/Two-photon_physics) im Detail beschrieben ist, unbedingt eine “Kollision” nennen willst und dich das glücklich macht, dann mach es doch einfach. In den Medien wird das sicherlich oft gemacht worden sein, weil es halt schön anschaulich klingt. Aber es ist trotzdem nicht das, was man normalerweise mit dem Wort “Kollision” bezeichnet und ich verstehe absolut nicht, warum es für dich so wichtig zu sein scheint, das unbedingt “Kollision” nennen zu dürfen.

  92. #92 stone1
    3. Februar 2014

    @Niels #81:
    …und fertig ist die Laube.
    Vielleicht unzulässig vereinfacht, dafür umso lässiger zu lesen. 😉

    Sphaleronen sind mir bisher auch noch nicht untergekommen, zumindest nicht bewusst. So einen seltsamen Namen hätte ich mir wohl gemerkt. Und das obwohl sie laut Wpedia schon 1984 erstmals beschrieben wurden. Irgendeine Chance, diese hypothetischen Teilchen am LHC experimentell nachzuweisen?

  93. #93 Alderamin
    3. Februar 2014

    @Lisa

    In dem von Dir verlinkten Golem-Artikel steht nichts von einer Teilchenerzeugung drin. Dafür wäre die Enerige der Photonen auch viel zu klein. Um ein Elektron-Positron-Paar zu erzeugen, braucht man harte Gamma-Strahlung, die so viel Enerige pro Photon hat, wie die Masse des erzeugten Teilchens (gemäß Einsteins berühmter Formel E=mc²).

    Die kann ein gewöhnlicher Laser nicht erzeugen (und es reicht auch nicht, mehr Laserstrahlung zu verwenden: die Energie eines einzelnen Photons kann nicht durch die vieler, weniger energiereicher Photonen ersetzt werden, siehe “Photoeffekt“).

    In dem Artikel steht nur drin, dass zwei Photonen sich unter gewissen Voraussetzungen wie ein Molekül verhalten können, will heißen, dass sie irgendwie aneinander gebunden bleiben, so wie zwei Wasserstoffatome ein Wasserstoffmolekül bilden. Das ist offenbar ein neuer Effekt. Aber sie bleiben Photonen. Bei der Kapitelüberschrift “Photonen bilden Moleküle” gehörte “Moleküle” eigentlich in Anführungszeichen.

  94. #94 Florian Freistetter
    3. Februar 2014

    @Alderamin: “In dem von Dir verlinkten Golem-Artikel steht nichts von einer Teilchenerzeugung drin. “

    Ne, das mit der Teilchenentstehung ist das andere Stanford-Dingens. Lisa spricht da immer abwechselnd von unterschiedlichen Artikeln…

  95. #95 Kallewirsch
    3. Februar 2014

    Lisa, ich hab irgendwie das Gefühl du machst den gleichen Fehler den ich auch lange Zeit machte (und wahrscheinlich immer noch mache). Du versuchst dein Wissen über unsere Welt ins Allerkleinste zu transportieren. Und genau da liegt der Hund begraben: das funktioniert so nicht. Kommt man mit unseren anschaulichen Allerweltsvorstellungen bei Atomen noch so einigermassen durch, so wird die Welt in noch kleineren Dimensionen immer bizarrer. Nichts, aber auch gar nichts, funktioniert auf dieser Ebene so, wie du es mit dem Hausverstand annehmen würdest, den du von Kind auf durch Beobachtung deiner Umgebung entwickelt hast.
    Und noch was: Bei der Beschreibung derartiger Vorgänge, ganz besonders in der nicht ganz so strengen Literatur, werdebn gerne Begriffe aus unserem Alltag benutzt. Die darfst du aber nicht allzu wörtlich nehmen. Das sind nur Analogien, damit man überhaupt einen Begriff hat und nichts neues erfinden muss, das dann erst recht wieder niemand versteht. Also die Dinge nicht allzu wörtlich nehmen. Wenn da von einer Kollision die Rede ist, dann sind das nicht 2 Kügelchen, die da aufeinander krachen.

  96. #96 Lisa
    3. Februar 2014

    @Florian,
    2 punkte…du schriebst:

    >>Ich habe gemeint: MEIN Artikel über instabile Photonen hat NICHTS mit irgendwelchen Experimenten in Stanford und auch NICHTS mit dem Golemartikel über “Lichtmoleküle” zu tun. <>Der Golemartikel ist außerdem sehr schlecht geschrieben. Allein der Satz “Als die Forscher aber zwei Photonen in die Rubidiumwolke schossen, passierte etwas Bemerkenswertes: Sie verließen die Wolke zusammen, !!!als ein einziges Molekül!!!.<>das sei ein neuer Materiezustand, sagen die Forscher.<>Die beiden Photonen zögen und schöben sich gegenseitig durch die Wolke.. “Es ist eine photonische Interaktion, die durch die atomare Interaktion herbeigeführt wird”, sagt Lukin. “Sie bringt diese beiden Photonen dazu, sich wie ein Molekül zu verhalten, !!!und wenn sie das Medium verlassen, dann zusammen und nicht als einzelne Photonen.!!! <<

    "Die atomare Interaktion führe dazu, dass sich die beiden Photonen WIE ein Moleküg verhalen…und das Medium haben sie interessanterweise nicht als einzelne Photonen, sondern gemeinsam verlassen."

    Ich suchte vorhin für dich 🙂 nach einem Vergleich dafür, dass es gar nicht so unpassend ist, wenn der Artikel es Molekül nennt, obwohl ein Molekül erst dann eins ist, wenn mindestens zwei elektrisch neutrale Teilchen gegeben sind, die aus zwei oder mehreren Atomen aufgebaut sind.

    Hier der Vergleich: Früher verstand man unter einer Ehe die Gemeinschaft von zwei unterschiedlichen Geschlechtern. Assoziation zu Molekül.

    Der "völlig neue Materiezustand", wie er im Artikel genannt wird, während die beiden Photonen diese Aktion hinlegten, also durch atomare Anregung zum gemeinschaftlichen Ziehen und Schieben gebracht werden,
    könnte man doch damit assoziieren, dass es sich bei der Eheschließung von Gleichgeschlechtlichen ebenfalls, ha, um einen völlig neuen Zustand handelt.
    Hier die "Gleichung", die ich erfunden habe:

    Altbekannter Zustand der zweigeschlechtlichen Ehe entspricht dem klassischen Molekül;
    neuer Zustand der gleichgeschlechtlichen Ehe entspricht
    dem "neuen Materiezustand", als was er in dem Artikel nicht unrichtigerweise beschrieben steht, finde ich.

    Molekül ist lediglich gem. herkömmlicher Definition nicht mehr passend, dem Sinn nach aber auch nicht wirklich unpassend. Oder?

  97. #97 Lisa
    3. Februar 2014

    @Aldemarin,
    wie Florian schon schreibt, meinte ich nicht den Golem-Artikel, sondern den anderen
    https://www.zeit.de/1997/42/materie.txt.19971010.xml

  98. #98 Lisa
    3. Februar 2014

    @Kallewirsch
    Okay, so wird es wohl wirklich nicht selten auch aus meiner Sicht bis jetzt ablaufen. Ich hoffe, dass ich halt nach und nach differenzieren lerne.

  99. #99 Florian Freistetter
    3. Februar 2014

    @Lisa: Molekül ist lediglich gem. herkömmlicher Definition nicht mehr passend, dem Sinn nach aber auch nicht wirklich unpassend. Oder?”

    Ohne unhöflich zu sein, aber: Darf ich dich nochmal fragen, was genau dein Anliegen ist? Geht es um MEINEN Artikel, den ich über die instabilen Photonen geschrieben habe? Möchtest du darüber diskutieren ob Photonen “kollidieren” können? Oder willst du über Licht”moleküle” und den korrekten Gebrauch von Vergleichen in der Wissenschaftskommunikation reden?

    Mir ist durchaus klar, warum das Wort “Molekül” in diesem Fall gewählt wurde, da brauchst du mir keine Vergleiche basteln. Aber es IST halt kein Molekül und man muss vorsichtig sein, wenn man solche irreführenden Vergleiche nutzt – genau so wie beim Wort “Kollision” wenn es um Quantenwechselwirkung von Photonenfeldern geht.

    Worum geht es dir denn jetzt eigentlich?

  100. #100 Lisa
    3. Februar 2014

    @Florian,
    du schriebst:
    >>Wenn du diese Art von Wechselwirkung, die im schon mehrmals erwähnten Wikipedia-Artikel (https://en.wikipedia.org/wiki/Two-photon_physics) im Detail beschrieben ist, unbedingt eine “Kollision” nennen willst und dich das glücklich macht, dann mach es doch einfach. In den Medien wird das sicherlich oft gemacht worden sein, weil es halt schön anschaulich klingt. Aber es ist trotzdem nicht das, was man normalerweise mit dem Wort “Kollision” bezeichnet und ich verstehe absolut nicht, warum es für dich so wichtig zu sein scheint, das unbedingt “Kollision” nennen zu dürfen.<>In Stanford dagegen wurden reale Lichtblitze aufeinandergeschossen. Mittels einer trickreichen Versuchsanordnung pumpten die Physiker dabei für eine billionstel Sekunde so viel Energie in einen milliardstel Quadratzentimeter, wie die gesamten Vereinigten Staaten in derselben Zeit verbrauchen. Eine der stärksten Laserquellen der Welt sandte extrem kurze Lichtpulse aus. Um einen zweiten, noch energiereicheren Strahl zu erzeugen, lenkten die Physiker dieses Laserlicht genau auf den feingebündelten Elektronenstrahl, der im Stanforder Beschleuniger erzeugt wird. Wie Pingpongbälle auf einen Lastwagen prallten die Lichtteilchen auf die Elektronen und wurden mit noch mächtigerem Stoß zurückgeworfen. Aus sichtbarem grünem Laserlicht wurde so höherenergetische Gammastrahlung, die mit dem nächsten Laserpuls kollidierte.<>Tatsächlich gleicht also das Vakuum in Wahrheit einem See, in dem unablässig Energie und Materie fluktuieren. Im Mittel addieren sich diese Fluktuationen stets zu null. Wenn allerdings in einem derart brodelnden Nichts zwei Lichtblitze mit Wucht zusammenprallen, dann kann diese Energie durchaus einem gerade fluktuierenden Partikelpaar zum Sprung von der möglichen in die reale Existenz verhelfen.<<

  101. #101 Lisa
    3. Februar 2014

    Du hast Recht, mir noch einmal die Frage zu stellen:

    >>Ohne unhöflich zu sein, aber: Darf ich dich nochmal fragen, was genau dein Anliegen ist? Geht es um MEINEN Artikel, den ich über die instabilen Photonen geschrieben habe? Möchtest du darüber diskutieren ob Photonen “kollidieren” können? Oder willst du über Licht”moleküle” und den korrekten Gebrauch von Vergleichen in der Wissenschaftskommunikation reden?<<

    Nein, es ging nachher nicht mehr um deinen Artikel selbst.
    Sondern um die Frage, ob Photonen durch Kollision Materieteilchen zu erzeugen in der Lage seien.
    Nicht unbedingt innerhalb Experimenten, die auf der Erde durchgeführt werden, sondern grundsätzlich im Universum, bzw. vor allem unmittelbar nach dem Urknall.

    Zur Falsifizierbarkeit einer Theorie ist es halt immer gut, auf real durchgeführte Experimente zu verweisen und dazu noch auf eindeutige Nachweise.
    Neals gab mir einerseits diese Bestätigung auch mit dem anderen von ihm oben angeführten Beitrag, der er damals schrieb. Doch du klangst für mich stets so, als haben Photonen nicht die Fähigkeit zur Materieerzeugung infolge von Kollisionen.
    Es geht um meine eigene Theorieentwicklung, für die ich diese durch Photonenkollisionen erzeugte Materie-Entstehung benötige. Oder infolge von Kollisonen der "harten" Photonen.
    Sorry bitte!!

  102. #102 Florian Freistetter
    3. Februar 2014

    @Lisa: Du hast den Artikel über das Experiment in Stanford jetzt schon oft genug zitiert. Ich weiss was da drin steht. Und ich hab dir jetzt auch schon mehr als einmal erklärt, was ich dazu zu erklären habe. Ich hab keine Lust, dass jetzt noch 5 Mal zu machen nur damit du mir dann noch 5 Mal irgendwas aus dem Artikel zitierst. Sag doch bitte einfach worum es dir geht und warum es so enorm wichtig für dich ist, dass du diesen physikalischen Effekt eine “Kollision” nennen darfst. Ich weiß derzeit echt nicht, was ich noch weiter erklären soll…

  103. #103 Lisa
    3. Februar 2014

    Die Beiträge kommen, weil ich manche Zeichen vermutlich falsch setze, manchmal anders, als ich sie geschrieben habe. Diese Zeichen z. B. habe ich eigentlich nie geschrieben.

    Der vorige Beitrag lautete ursprünglich anders, wahrscheinlich hatte ich etwas anderes aus der Datei, in die ich zuerst schrieb, kopiert und hier eingefügt.

    Da fragte ich jedenfalls, ob überhaupt das, was Wikipedia in de Two Photonen-Beitrag schreibt, das Gleiche ist, was in Stanford gemacht wurde, weil es dort mit enormer Wucht passierte, also tatsächlich innerhalb einer regelrechten Kollision.
    Hier ist der Artikel, er besteht aus 2 Seiten.
    https://www.zeit.de/1997/42/materie.txt.19971010.xml/seite-2

  104. #104 Lisa
    3. Februar 2014

    Sorry Florian,
    mein Beitrag hat sich mit deinem überschnitten.

    Eigentlich frage ich dich das ja nicht, sondern rege dich an, den Artikel vielleicht neu zu lesen, also dort, wo es eindeutig um Kollision geht. Ich nehme an, es besteht auf deiner Seite ein Missverständnis, weil du davon ausgehst,
    in Stanford sei das Geliche gemacht worden, wie das, was Wikipedia schreibt.

    Deine Argumente passen m. E. nicht 100 %ig, also es sieht für mich so aus, als habest du, auch trotz 5-maligem Posten der URL den gesamten Artikel nicht so gelesen, dass auch du merkst, da handelte es sich sehr wohl um knallharte Kollision von Photonen.

  105. #105 Florian Freistetter
    3. Februar 2014

    @Lisa: “dass auch du merkst, da handelte es sich sehr wohl um knallharte Kollision von Photonen.”

    Ok, ich gebe die Diskussion auf. Wenn es für dich so enorm wichtig ist, dass Photonen “knallhart” kollidieren können, dann glaub bitte daran. Ich weiß zwar nicht, warum das so wichtig für dich ist und warum du nicht verraten willst, worum es dir geht, aber das ist auch egal…

    (P.S. Wenn ich wirklich vernünftig beurteilen soll, was da gemacht worden ist, dann musst du mir die Primärquelle zeigen, also die wissenschaftliche Facharbeit und nicht das, was irgendein Journalist in einer deutschen Zeitung darüber geschrieben hat. Es mag zwar überraschend klingen, aber: Manchmal haben Journalisten keine Ahnung und schreiben Dinge, die nicht richtig sind…)

    (P.P.S. Hier ist eine Liste mit wissenschaftlichen Artikeln die zu dem Stanford-Experiment veröffentlicht worden sind: https://www.slac.stanford.edu/exp/e144/papers.html Der Begriff den du dort lesen kannst heißt “Light-by-Light Scattering” und das ist genau das, was in der Wikipedia steht und genau das, was ich dir dir schon x-Mal gesagt habe: Eine Wechselwirkung zwischen Photonen, die Quantenfelder anregt aus denen dann neue Teilchen gemäß E=mc² erzeugt werden. Aber ich nehme an, du wirst weiterhin darauf bestehen, dass ich mich irre und weiterhin nur den Journalisten der Zeit als einzigen Experten gelten lassen…)

  106. #106 Florian Freistetter
    3. Februar 2014

    @Lisa: “Es geht um meine eigene Theorieentwicklung, für die ich diese durch Photonenkollisionen erzeugte Materie-Entstehung benötige. Oder infolge von Kollisonen der “harten” Photonen.”

    Sorry, das hatte ich vorhin übersehen.

    Ok, jetzt ist mir auch klar worum es geht. Deine private Theorie – ok… sag mir Bescheid, wenn du sie in einer wissenschaftlichen Fachzeitschrift publiziert hast, dann schau ich mir das gerne an. Bis dahin kann ich dir leider nicht weiter helfen.

  107. #107 Niels
    3. Februar 2014

    @stone1

    Irgendeine Chance, diese hypothetischen Teilchen am LHC experimentell nachzuweisen?

    Sorry, keine Ahnung.

    @Lisa

    So denke ich noch immer, dass Niels und du nicht das gleiche meint.

    Doch, das tun wir.

    da handelte es sich sehr wohl um knallharte Kollision von Photonen

    Was meinst du genau mit “knallharter Kollision”?
    Es geht um einen quantenmechanischen Vorgang, der sich auch nur mit Mitteln der Quantenmechanik beschreiben lässt. Das nennt man üblicherweise Wechselwirkung oder Streuung.
    Es kommt aber durchaus manchmal vor, dass das Kollision genannt wird.
    https://scholar.google.de/scholar?q=photon+photon+collision&btnG=&hl=de&as_sdt=0%2C5
    Dir muss nur klar sein, dass das nichts mit einem Stoß zu tun hat, wie er in der klassischen Physik vorkommt.

    @Florian
    Ah, danke. Es ging also um das hier:

    These results are the first laboratory evidence for inelastic light-by-light scattering involving only real photons.

    https://www.hep.princeton.edu/~mcdonald/e144/burke_prl_79_1626_97.pdf

  108. #108 JaJoHa
    3. Februar 2014

    @Lisa
    Schau mal hier, da geht es um das Erzeugen von Gammastrahlung mit einem Elektronenstrahl und Laserlicht.
    Das im Artikel angesprochene Experiment scheint die Bezeichnung E144 zu haben und ließ die Photonen mit einem 50 GeV Elektronenstrahl wechselwirken. Hier wird anscheinend auch genauer erklärt was das sollte und der Aufbau etc. Da ging es anscheinend vorallem um das testen der QED.

    Übrigens vorsichtig sein bei “hart” im Zusammenhang mit Strahlung. “Harte Gammastrahlung” bedeutet hohe Photonenenergie.

  109. #109 Lisa
    3. Februar 2014

    @Florian und @Niels,

    das, was ihr bezüglich des Wikipedia-Artikels erklärt, dass es sich dabei um keine heftige Kollision handelt wie ich stets zu glauben geneigt bin, glaube ich euch ja grundsätzlich.
    Doch wo ich noch nicht sicher bin, ist ob es sich bei dem in Wikipedia Beschriebenen um das Gleiche Experiment handelt wie das aus Stanford. Allein um die Frage geht es mir doch.
    Hier habe ich übrigens einen weiteren Link gefunden, den jemand aus einem anderen Forum gepostet hatte, also nicht an mich, ich fand ihn lediglich und beachtet bitte die Begriffe “unglaublicher Wucht”, “1000 Milliarden Watt Leistung”, “fast mit Lichtgeschwindigkeit” , “extrem energiereiche Gammastrahlung” und “insgesamt gelang dies hundertmal”
    Stimmt dies alles mit dem aus Wikipedia Beschriebenen überein? Ja oder nein?
    Und hier der kurze Artikel:
    https://www.zeit.de/1997/41/Materie_aus_Licht
    “…
    …… Nun schaffte es das US-Team unter Führung von Adrian Melissinos, Licht in Partikel umzusetzen, in Elektronen und Positronen.
    Sie nutzten dazu eng gebündeltes grünes Licht eines Lasers mit 1000 Milliarden Watt Leistung und schossen es gegen Elektronen, die fast mit Lichtgeschwindigkeit flogen.
    Diese Frontalkollision verglichen die Physiker mit “einem Pingpongball, der auf einen rasenden Lastwagen prallt”. Der Stoß schleudert das Photon mit unglaublicher Wucht zurück.
    Dabei entsteht extrem energiereiche Gammastrahlung, die ihrerseits kollidiert mit nachfolgenden Photonen im Laserstrahl.
    Daraus entstehen ein Elektron und ein Positron. Das Positron läßt sich nachweisen, insgesamt gelang dies hundertmal.”
    ———————-
    Der von euch jeweils genannte Wikipedia-Artikel beschreibt m. E. ein anderes Experiment.
    Bei dem von mir immer wieder erwähnten aus Stanford fanden nunmal echte Kollisionen mit hoher Wucht statt, m. E.
    Bei dem von Euch erwähnten nicht. So jedenfalls verstehe ich es bis jetzt.
    Aus euren Antworten kann ich kein mich überzeugendes Argument lesen, sondern nur eure Vermutung, dass es sich biem Stanford-Experiment um dasselbe wie in Wikipedia beschriebene handeln würde.
    Kann sein, kann nicht sein. Hierüber hätte ich lediglich gerne Klarheit und nicht allein eure Vermutung.
    Konnte ich jetzt den Unterschied, um den es mir geht, heraus stellen, besser?
    Ich meine, wir reden hier lediglich aneinander vorbei. Ihr habt Recht, falls Stanford dem von Wikepedia entspricht.
    Aber nicht habt ihr evtl. Recht, falls es zwei unterschiedliche Experimente waren und in Stanford ganz einfach echte Kollisionen stattfanden. Ich hatte lediglich gehofft, jemand wisse darüber genau Bescheid oder könne es heraus finden.

  110. #110 Florian Freistetter
    3. Februar 2014

    @Lisa: “Doch wo ich noch nicht sicher bin, ist ob es sich bei dem in Wikipedia Beschriebenen um das Gleiche Experiment handelt wie das aus Stanford. Allein um die Frage geht es mir doch.”

    Offensichtlich nicht. Denn diese Frage wurde schon mehr als einmal beantwortet: Ja, es ist das gleiche. Dir geht es offensichtlich nicht um die Antwort, sondern um eine Antwort die dir gefällt damit sie zu deiner Privattheorie passt. Die kann ich dir aber nicht liefern. Sorry.

    Und BITTE hör auf ständig aus diesem Zeit-Artikel zu zitieren. Wir haben ihn mittlerweile alle gelesen. Wenn du wissen willst, was wirklich passiert ist, lies die Fachartikel, die ich dir verlinkt habe. Wenn du tatsächlich Wissenschaftler sein willst, musst du Primärquellen lesen und nicht irgendwas was irgendein Journalist geschrieben hat.

    “Hierüber hätte ich lediglich gerne Klarheit und nicht allein eure Vermutung.”

    Lies die Fachartikel. Nochmal: Lies die Fachartikel. Das, was irgendein deutscher Journalist über ein wissenschaftliches Experiment schreibt ist absolut irrelevant. Lies die Fachartikel. Da steht drin, was wirklich passiert ist. So läuft WIssenschaft. Du willst deine eigene wissenschaftliche Theorie schaffen? Dann musst du dafür auch selbst arbeiten. Das kann dir niemand abnehmen. Und wie man sieht, nützt es ja sowieso nichts, weil du nicht akzeptieren willst, dass bei dem Stanford-Experiment nichts “kollidiert” ist (was die diversen Fachartikel die ich verlinkt habe, wiederlegen). Wenn du nicht bereit bist, die Realität zu akzeptieren und nicht bereit bist, selbst zu denken, dann wird das nix mit der neuen wissenschaftlichen Theorie…

  111. #111 Niels
    3. Februar 2014

    @Lisa

    Der von euch jeweils genannte Wikipedia-Artikel beschreibt m. E. ein anderes Experiment.

    Nein.
    Die verschiedenen Wikipedia-Artikel beschreiben nämlich eigentlich gar keine Experimente. Sie beschreiben vielmehr den Prozess bzw. die Physik, die diesem Experiment zugrunde liegt und die damit untersucht wurde.

    In Stanford wurde Licht an Licht gestreut und die dabei entstehenden Materieteilchen detektiert. Wie das Experiment dabei genau aufgebaut war ist für das Verständnis das grundlegenden quantenmechanischen Prinzips völlig unerheblich.
    Es ist unwichtig, ob das gebündeltes grünes Licht eines Lasers mit 1000 Milliarden Watt Leistung beteiligt war. Es spielt keine Rolle, dass extrem energiereiche Gammastrahlung erzeugt wurde. Es ist völlig egal, wie diese Gammastrahlung erzeugt wurde (nämlich durch die Streuung des Laserlichts an hochenergetischen Elektronen, also durch den inversen Comptoneffekt.)
    Für das Verständnis des Ganzen ist nur wichtig, dass die Photonen irgendwie genügend Energie zur Verfügung haben, damit nach E=mc^2 die Schwellenenergie zur Erzeugung eines Teilchen-Antiteilchenpaar erreicht wird.
    Deswegen ist die Aufzählung all dieser Superlative im Zeit-Artikel ziemlich sinnlos. Auch wenn das für den Autor anscheinend das Wichtigste war.

  112. #112 Lisa
    4. Februar 2014

    @Niels und Florian,

    Hand aufs Herz, habt ihr den Stanford-Artikel Wort für Wort auf beiden Internet-Seiten gelesen? Wenn nicht, dann tut es bitte!
    Vor allem die zweite Seite machts deutlich, dass es sich um ein einzigartiges Experiment handelt, das nicht mit dem von euch vorgeschlagenen zu vergleichen ist..
    Der Wikipedia-Artikel erwähnt mit keinem Wort Stanford oder diese 20 Wissenschaftler oder auch nicht Adrian C. Melissinos.

  113. #113 Lisa
    4. Februar 2014

    Mein voriger Beitrag ist dann nicht mehr relevant, vergisst ihn bitte wieder. Ich akzeptiere, was du, Niels, mir erklärt hast.
    Woran ich mich noch störe ist, dass Florian meint:
    “Aber es ist trotzdem nicht das, was man normalerweise mit dem Wort “Kollision” bezeichnet””

    Mein Partner meinte vorhin, dass in Stanford wegen der verschiedenen Steigerungen extrem starke Kollisionen statt fanden, und daher verstehe ich nicht, weshalb Florian das so anders sieht oder auch du.

    Vielleicht verstehe ich es irgendwann einmal.
    Vielleicht aber gibt es auch einiges, das du und Florian nicht versteht. Damit meinen wir speziell den 6. Abschnitt, wo in mehreren Stufen jeweils Verstärkung zu den Verstärkungen erzeugt wurden.

    “Mittels einer trickreichen Versuchsanordnung pumpten die Physiker dabei für eine billionstel Sekunde so viel Energie in einen milliardstel Quadratzentimeter, wie die gesamten Vereinigten Staaten in derselben Zeit verbrauchen.

    Eine der stärksten Laserquellen der Welt sandte extrem kurze Lichtpulse aus.

    Um einen zweiten, noch energiereicheren Strahl zu erzeugen, lenkten die Physiker dieses Laserlicht genau auf den feingebündelten Elektronenstrahl, der im Stanforder Beschleuniger erzeugt wird. …..

    prallten die Lichtteilchen auf die Elektronen und wurden mit !!!noch mächtigerem Stoß zurückgeworfen!!!.

    Aus sichtbarem grünem Laserlicht wurde so höherenergetische Gammastrahlung, die mit dem nächsten Laserpuls kollidierte.”

    Hier passt das Wort Kollisionen doch definitiv, während Niels sagt, es passt nicht wirklich.
    Und so etwas verstehe ich nicht bis jetzt, und schließe daraus, dass er nicht die Vorgänge in Stanford meinen kann, sondern das in Wikipedia beschriebene Prinzip.

    Ich will nicht nerven. Daher brauche ich, wenn ihr nicht mehr mögt, keine weitere Antwort. Wünschen würde ich mir sie, doch es macht nichts, wenn ihr es hier stehen lassen wollt.
    Dann gehe ich jetzt mal schlafen und sage Allen Gutnacht und danke.

  114. #114 Lisa
    4. Februar 2014

    @ JaJoHa
    Danke für deine Hinweise. Ich versuche sie morgen zu lesen und zu vergleichen…

  115. #115 Lisa
    4. Februar 2014

    Obendrüber verschrieb ich mich und meinte anstatt “Niels” “Florian”

  116. #116 Florian Freistetter
    4. Februar 2014

    @LIsa: Ich habe dem bisher gesagten nichts mehr hinzuzufügen. Du bestehst weiter darauf, dass der Artikel eines deutschen Journalisten ÜBER ein Experiment relevanter ist als die von dem am Experiment beteiligten Forscher selbst verfassten Fachtartikel. Du versteifst dich weiter aus völlig schleierhaften und absurden Gründen darauf, ein ganz bestimmtes Wort verwenden zu dürfen, was völlig irrelevant ist. Sag meinetwegen, dass sich die Photonen “geküsst” haben und so neue Teilchen erzeugt haben. Sag, dass eine “Zerlopation” stattgefunden hat. Oder denk dir selbst Wörter aus. Es ist völlig schnurz. Es gab eine Wechselwirkung zwischen Photonen und in der Physik wird dieser Prozess “Streuung” genannt. Wenn du das unbedingt “Kollision” nennen willst, dann mach es doch einfach und mach hier nicht so ein gewaltiges Drama…

    Und wenn du selbst unbedingt Wissenschaft betreiben willst, dann wirst du nicht umhin können, dass zu tun, was Wissenschaftler eben tun: hart arbeiten, die Mathematik lernen und die Primärquellen lesen. Mit der Lektüre von irgendwelchen Zeitungsartikel revolutioniert man die Teilchenphysik nicht und man erschafft damit auch keine Alternative zur Urknalltheorie (vermutlich gehts ja wieder mal darum…)

  117. #117 Niels
    4. Februar 2014

    @Lisa
    Ich verstehe dein Problem nicht ganz

    Licht besteht eben nicht aus kleinen Pingpongbällen, egal was in diesem Zeit-Artikel steht.
    Beim inversen Comptoneffekt, also der der Streuung des Laserslichts an den Elektronen, macht eine Veranschaulichung mit Hilfe kleiner Pingpongbälle ja noch irgendwie Sinn.
    Bei der Streuung von Licht an Licht versagt dieses Bild aber völlig. Deswegen ist es nicht unbedingt hilfreich, wenn man dabei das Wort “Kollision” benutzt.
    Vielleicht schaust du für den Anfang erst mal hier rein.
    Die Beschäftigung mit diesem speziellen Experiment ist dabei ganz bestimmt kein guter Einstieg, um ein erstes Verständnis der Elementarteilchenphysik zu bekommen. Da würde ich an deiner Stelle ganz langsam mit einigen populärwissenschaftlichen Büchern anfangen, die sich mit den Grundlagen beschäftigen.
    Hier kann dir bestimmt jemand etwas Aktuelles auf Einstiegsniveau empfehlen.

    Außerdem geht es hier um Hochenergiephysik. Der Name verrät schon, dass man bei diesem Gebiet immer mit irgendwelchen Superlativen um sich werfen kann.

    Zum Thema:
    Vielleicht liest du dir Florians und meine Antworten noch einmal der Reihe nach durch? Ich finde keine neue Frage, das wurde doch alles schon beantwortet.

  118. #118 Lisa
    4. Februar 2014

    @Florian,
    meine Fragen an dich waren gleichzeitig Anstöße. Diese geben sich alle wissenschaftlich ausgerichteten Menschen hier und da. Ich nehme viele an und konnte viele schon an Andere weiter geben.

    …dabei ging es die ganze Zeit um nur 2 Fragen, die bis jetzt, wenn auch gut gemeint von euch und ich mich über euer Antworten gefreut habe, dennoch unbeantwortet blieben.

    @Florian und @Niels,

    Frage 1
    Das was ich quasi gerne gewusst hätte, ist, um welchen der drei Interaktionsprozesse beim “Two-photon physics” oder auch “gamma–gamma physics” genannt, es sich eurer Meinung nach im Stanford-Experiment gehandelt hatte.

    Denn in dem Wikipedia-Beitrag sind diese drei genannt:
    1.
    Direct or pointlike: The photon couples directly to a quark inside the target photon.[2] If a lepton–antilepton pair is created, this process involves only quantum electrodynamics (QED), but if a quark–antiquark pair is created, it involves both QED and perturbative quantum chromodynamics (QCD).[3] [4] The intrinsic quark content of the photon is described by the photon structure function, experimentally analyzed in deep-inelastic electron–photon scattering.
    2.
    Single resolved: The quark pair of the target photon form a vector meson. The probing photon couples to a constituent of this meson.
    3.
    Double resolved: Both target and probe photon have formed a vector meson. This results in an interaction between two hadrons.”
    Quelle: https://en.wikipedia.org/wiki/Two-photon_physics

    (Ins vom Englischen ins Deutsche übersetzt heißt es dann noch):
    “Für die beiden letztgenannten Fällen ist das Ausmaß der Wechselwirkung wie der starke Kopplungskonstante groß ist. Dies wird als Vektor-Meson-Dominanz (VMD) und hat in nicht-störungstheoretischen QCD modelliert werden.” Zitatende
    Quelle:
    https://translate.google.de/translate?hl=de&sl=en&u=https://en.wikipedia.org/wiki/Two-photon_physics&prev=/search?q%3DTwo-photon%2Bphysics%26biw%3D1280%26bih%3D878

    Welche der drei Prozesse lief in Stanford ab?
    Denn davon würde ich es evtl. abhängig machen, ob ich das Wort “Kollision” oder das Wort “Streuung” verwenden werde.
    —————

    Evt.. einer der letzten beiden?? Denn angesichts der letzten beiden heißt es, -wegen des Begriffs “Kollision” – ja:
    “Für die beiden letztgenannten Fällen ist das Ausmaß der Wechselwirkung wie der starke Kopplungskonstante groß ist. Dies wird als Vektor-Meson-Dominanz (VMD)” …usw.

    Da hatte ich gehofft, dass hier auf Stanford bezogen das Wort “Kollision” sehr wohl passt, weil vor allem du, Florian, stets betonst, dass unter Kollision EIGENTLICH etwas anderes zu verstehen sei, ..aber gnädigerweise könnte ich es auch gerne so oder so nennen.
    Genau da möchte ich wissen, ob, wie ich vermute, tatsächlich sogar die meisten Wissenschaftler im Gegensatz zu dir, den Begriff “Kollision” als am ehesten korrekt erachten, da der Interaktionsprozess einer der beiden letzten der drei oben genannten war.
    ————————-
    War es einer der beiden letzten oder war es der erste in Wikipedia genannte Interaktionsprozess?
    Auf den ersten bezogen könnte ja dein stetiges Daraufhinweisen, dass der Begriff “Kollision” nicht passend ist, korrekt sein, dann würde ich auch das Wort “Streuung” von dir übernehmen können oder wollen.
    —————————-

    Während bei den beiden letzten Interaktionsprozessen das Wort “Kollision” evtl. sehr wohl passend ist und kein anderes.

    Doch hole mal egal welchen Bericht hervor, wo es um Teilchenbeschleuniger geht, immer heißt es da “Kollision”.
    Egal, um welche Teilchen es sich handelt oder “nur” um masselose Teilchen.
    Nur, weil das adäquate Verb bei Photonen-Interaktion laut Lehrbuch “streuen” heißt, verhältst du dich also dermaßen treu und mir gegenüber dermaßen gnädig. Und gnädig ist niemals Augenhöhe. Wundere dich nicht, wenn Gnädig-Getue provozierend rüber kommt.

    Glaubst du wirklich bei diesen Geschwindigkeiten mit zudem dieser Frequenzverschiebung, um die Photonen noch energiereicher zu machen, würde noch das zarte Wort “streuen” passen”. Das ist ein knallharter krasser Zusammenprall und sonst nichts.
    Deshalb nennt auch der Artikel das Kind beim Namen, wenn von Kollision die Rede ist.

    Und wie meine 2. Frage etwas weiter unten lautet: Ab wann ist bei dir etwas eine Kollision? Denn ich möchte nicht, so wie du es rüber bringst “gnädigerweise” ja auch gerne das Wort “Kollision” verwenden dürfen, wenn es aber doch in Wirklichkeit allgemein das korrekte Wort ist, wenn Teilchen, zu denen in dem Fall auch Photonen gehörten, in Teilchenbeschleunigern aufeinander zu knallen.
    Streuen kann man Salz im Winter und Photonen an Feinstaub usw. aber doch nicht in Teilchenbeschleunigern.

    Doch nicht mehr, wenn es um derartiges Aufeinanderprallen geht.
    ————–
    ————–
    Meine bisherigen Fragen waren eher Anstöße für dich. Ich zog halt dennoch in Erwägung, dass du trotzdem Recht haben könntest und „Streuung“ richtig sein könnte.
    Doch nun schaute ich mir viele Artikel von Physikern an und überall nennen die es auch “Kollision”.
    ————–

    Frage 2
    Ab wann würde das Wort “Kollision” passen, wenn nicht bei dem was in Stanford ablief.
    Im Internet wird das Aufeinanderprallen von Teilchen, wenn es innerhalb von Teilchbeschleunigern geht, überall “Kollision” genannt.

    In Wikipedia entspricht “elastische Streuung” dem “elastischen Stoß”; “Stoß” wiederum entspricht “Kollision”.
    So passt laut Lehrbuch also sowohl Stoß, Streuung als auch Kollision.
    Jetzt zum authentischen Punkt:
    Da im Stanford Teilchenbeschleuniger masselose Photonen gleich wie Teilchen behandelt wurden, gelten auch hier die typischen Interaktionsprozess-Begriffe wie bei den bisherigen Teilchen in Beschleunigern, nämlich da heißt es immer “Teilchen-Kollision”.
    Die Photonen wurden nunmal zudem unter zusätzlicher Frequenzverschiebung, um sie noch energiereicher zu machen, voll!!! auf Kollisionskurs!!! gebracht, um sie miteinander kollidieren zu lassen, und wie heftig sogar! Sagten die dort etwa: „He Mann, kommt, lasst uns mal Photonen aneinander streuen!“
    Und du hältst das zarte Wörtchen “Streuung” also für eher angebracht? Und gestattest mir gnädigerweise den angeblich unpassenden Ausdruck „Kollision“?

    Ich fühlte mich dadurch provoziert und möchte dich nicht zurück provozieren, nur dir vermitteln, worum es geht, und was glaube ich tatsächlich so ist.
    Das ist für mich sehr wichtig und keine “Wortspalterei”.
    Wegen meiner Theorieformulierung, will da keinen unnötigen Fehler machen, auch darin knallen Photonen – jedoch nicht nur aus läppischen zwei Richtungen kommend – mit Lichtgeschwindigkeit aufeinander zu. Sie kommen aus so vielen Richtungen, wie es Pixel oder Punkte auf einer Kugeloberfläche von der und der Größe gibt und „treffen“ sich alle frontal in der Mitte. Da ist jeder Teilchenbeschleuniger auf der Erde glatt ein Spielzeug dagegen.

  119. #119 Kallewirsch
    4. Februar 2014

    Dann nenn es halt Kollision.
    Wenn es dir darum geht: Du kriegst den Sanktus dazu.

    Das ist für mich sehr wichtig und keine “Wortspalterei”.

    Das ist völlig unwichtig, ob du es Kollision oder Streuung nennst. Denn es ist weder das eine noch das andere. Je nach Betrachtungsweise.
    Ja, das ist das hinterhältige an der Quantenwelt. Die Objekte sind weder Teilchen noch Wellen. Und doch auch wieder schon. Je nachdem wer hinsieht und welches Experiment er macht. Hier bricht ganz einfach jegliche Anschaulichkeit zusammen, die wir aus unserer Makrowelt mitbringen. Kollision ist es für mich, wenn ein Auto gegen einen Baum kracht. Das aber passiert so nicht, denn weder Elektron noch Photon sind kleine feste Kügelchen, die aneinander prallen können.

    Ist das so schwer zu verstehen, dass hier all unsere Alltagsbegriffe komplett versagen, wenn man genauer hinsieht? Man kann die Begriffe maximal je nachdem über welches Experiment man gerade redet, so ungefähr verwenden, weil es vage Analogien gibt. Aber im Detail darf man eben nicht hinsehen, denn dann ist im Grunde alles falsch, was du an Begriffen aus deiner Alltagswelt benutzen willst.

    Wegen meiner Theorieformulierung

    Oh, bitte. Auf dieser Ebene werden Theorien nicht formuliert, indem man sich irgendwelche schönen Bilder ausdenkt, mit denen man Laien beeindrucken kann. Auf dieser Ebene der Materie (also weit unter Atomgröße) sind Theorien knallharte Mathe. Denn alles andere ist komplett missverständlich. Unsere übliche Wortwahl bricht zusammen. Photonen kollidieren nun mal nicht mit sich selbst oder mit Elektronen. Auch wenn es in manchen Experimenten vielleicht auf den ersten Blick so aussehen mag. Und sie streuen auch nicht aneinander, auch wenn es auf den ersten Blick in manchen Experimenten auf den ersten Blick so aussehen mag. Genau das ist es nämlich, was Quantentheorie so schwer, aber auch so spannend macht. Wir können uns nur 2 Arten von Objekten vorstellen: Teilchen oder Wellen. Beide sind inkompatibel zueinander. Wenn etwas ein Teilchen ist, dann kann es keine Welle sein. Und umgekehrt. Und trotzdem passiert das unfassbare: Elementarteilchen zeigen beide Verhaltensmuster, je nachdem bei welchem Experimentz man hinsieht. Sie sind also weder Teilchen noch sind sie Wellen, sondern .. ja was eigentlich? Sie sind Objekte, die durch die Gleichungen der Quantentheorie beschrieben werden können. Unsere alltäglichen Worte greifen hier einfach nicht mehr.

  120. #120 JaJoHa
    4. Februar 2014

    @Lisa
    Zu 1
    Die Prozesse die erlaubt sind laufen alle ab, du kannst leider nicht sagen “Jetzt bitte nur Prozess X”. Hier siehst du das ganze für LEP, die Daten sind vom L3. Aber da sind nur die intressanten Prozesse aufgelistet, parrallel laufen da auch Paarproduktionen von allen Quarks (außer t), alle Leptonen und alles was du daraus zusammenbauen kannst (also Mesonen, Baryonen).
    Deshalb ist die Frage 1,2 oder 3 so nicht zu beantworten, bzw die Antwort wird wenn in die Richtung X%, Y%, Z% gehen. Das ist schon bei Elektron-Positron der Fall, kann über Z-Boson oder übers Photon und dazu die Auswahl zwischen Streuung und Annihilation. Du kannst teilweise die einzelnen Beiträge berechnen und das auch “sehen” (z.B. Änderungen in der Verteilung, Wirkungsquerschnitt etc).

    Zu 2.
    Bezeichnungen sind relativ egal, wichtig ist aber das du dir die Kollision nicht wie zwischen zwei harten Kugeln vorstellen darfst. Hier siehst du wo sich Elektronen beim Wasserstoffatom aufhalten, bedenke dabei das Integral über den gesamten Raum ist 1, aber an jedem Punkt ist das 0.

  121. #121 silur
    18. Oktober 2014

    dieser Thread ist zwar schon sehr alt, aber vielleicht kann mir jemand dennoch eine Antwort auf die Frage geben, die mir dabei eingefallen ist:

    Wie funktioniert “Lichtdruck”?
    Wenn ich davon ausgehe, dass da eine Kraft auf zb. ein Sonnensegel ausgeübt wird, dann gilt doch Kraft = Masse x Beschleunigung
    Also das klingt doch danach, dass das Licht, das diese Kraft auf das Sonnensegel bewirkt eine Masse haben müsste…?

    sorry für meine “Küchenphysik” – ich denke, dass die Erklärung irgendwie anders sein muss…
    ..aber wie?

  122. #122 Florian Freistetter
    18. Oktober 2014

    @silur: “dass das Licht, das diese Kraft auf das Sonnensegel bewirkt eine Masse haben müsste…?”

    Das wichtige Wort ist hier nicht “Masse” sondern “Impuls”. Es wird ein Impuls übertragen… https://scienceblogs.de/astrodicticum-simplex/2009/11/16/segeln-im-weltall/

  123. #123 silur
    18. Oktober 2014

    @Florian:
    Mmmhhh sorry, immer noch unklar….. auch in deinem verlinkten Artikel gibts einen weiteren Link unter “Impuls” wo dieser als Produkt von Masse und Geschwindigkeit definiert ist.
    Auch Wikipedia: cit:
    “Die physikalische Größe Impuls, auch als Bewegungsgröße oder Bewegungsmenge bezeichnet, beschreibt die Bewegung eines massebehafteten Körpers.”

    …sorry, ich will nicht lästig sein, bin auch kein eso-spinner, nur ein interessierter Bio-Lehrer..

  124. #124 rolak
    18. Oktober 2014

    Auch Wikipedia

    Ja, silur, doch schon etwas weiter hinten wird auch der Impuls eines Photons angegeben.

    lästig

    Das wird durch Fragen sicherlich nicht ausgelöst werden, schlimmstenfalls irgendwann Gegenfragen nach der Motivation, dem Ziel – um zu ergründen, ob denn bis dahin überhaupt über die richtigen Fragen nachgedacht wurde.

  125. #125 silur
    18. Oktober 2014

    @rolak: … ah ja.. zugegebenermaßen hab ich da nicht weiter gelesen, geschweige denn dass ich das Formelmeer durchblickt hätte.
    Aber ich akzeptiere gern, dass Photonen einen Impuls aber dennoch keine Masse haben!
    vielen Dank!

  126. #126 Florian Freistetter
    18. Oktober 2014

    @silur: Hilft es vielleicht wenn ich sage, das Licht keine RUHEmasse hat. Aber Licht ruht ja nicht, sondern bewegt sich sehr schnell und hat deswegen durchaus Impuls. Und wenn der sich bei Reflektion von -p auf +p ändert, wurden dazwischen 2p übertragen. Und wie Newton schon wusste: Jede Änderung eines Impuls entspricht einer Kraft…

  127. #127 JaJoHa
    19. Oktober 2014

    @silur
    Man kann den Lichtdruck verschieden beschreiben, auch über die Maxwell-Gleichungen.
    Und das man für die Masse des Photons 0 einsetzt ist dadurch legitimiert, das man die Masse zu klein ist um sie zu messen. Falls das Photon eine Ruhemasse hätte, dann müsste sie kleiner als 10^{-18}eV/c^2\approx 10^{-53}kg sein.

  128. #128 Peter
    24. August 2018

    Nach Albert Einsteins Formel E=mc² sind ja Masse und Energie äquivalent. Da die Lichtgeschwindigkeit c eine Konstante ist, müsste man ja die theoretische Masse eines Photons aus seiner Energie berechnen können.
    Dann müssten aber energiereiche Photonen (z. B. Photonen der Gammastrahlung) mehr (theoretische) Masse haben als Photonen mit mittlerer Energie wie z. B. die des sichtbaren Lichts, welche wiederum mehr (theoretische) Masse haben müssten als energieärmere Photonen z. B. von Radiowellen.
    Könnte die größere Trägheit von massereicheren Photonen erklären, warum Gammastrahlung tiefer in dichte Objekte eindringt als Röntgenstrahlung, diese tiefer eindringt als UV-Strahlung und diese tiefer eindringt als sichtbares Licht?

    Wenn Photonen eine – wenn auch sehr geringe – Masse haben, würde dies erklären, warum Photonen durch starke Massen abgelenkt werden (Gravitationslinseneffekt)?

    Oder könnte man, wenn Photonen masselos sind, die Ablenkung von Photonen durch starke Massen allein dadurch erklären, dass starke Massen ja die Raumweit krümmen?

  129. #129 Bullet
    24. August 2018

    @Peter:

    müsste man ja die theoretische Masse eines Photons aus seiner Energie berechnen können.

    Jepp.

    Dann müssten aber energiereiche Photonen…

    Jepp.

    Könnte die größere Trägheit von massereicheren Photonen erklären, warum Gammastrahlung tiefer in dichte Objekte eindringt als Röntgenstrahlung

    Nope. Sie können aber besser ionisieren. Im Sinne von: sie können besser und mehr Elektronen aus Atomhüllen rauskloppen oder sogar den Atomkern spalten. (Deutsche Wikipedia -> “Kernphotoeffekt”, englische Wikipedia -> “photofission” – Vorsicht: das sind zwei verschiedene Effekte!)

    Wenn Photonen eine – wenn auch sehr geringe – Masse haben, würde dies erklären, warum Photonen durch starke Massen abgelenkt werden (Gravitationslinseneffekt)?

    Nö. Sie haben eben keine “Masse”. Nur den Energiegehalt. Und Masse zieht nicht am Photon, sondern verbiegt den Raum darunter, so daß das Photon “glaubt”, geradeaus zu fliegen, aber in Wirklichkeit abgelenkt wird. Das ist dann deine zweite Möglichkeit.